Pediatrics Case Files

¡Supera tus tareas y exámenes ahora con Quizwiz!

In DUB (dysfunctional uterine bleeding), after verifying the patient is not pregnant, what is the next important test to get to help guide management?

- Hemoglobin, as it helps categorize the severity of bleeding and helps guide future management. - A Hb <12 is mild, 9-12 is moderate (Treat with iron and OCP), and < 9 is severe (hospitalize & transfuse with IV estrogen and high dose OCPs until the bleeding stops).

CLUBBING:

- Increase in the angle between the nail and nail base of 180° or greater, and softening of the nail base to palpation. - Although the condition can be familial, clubbing is uncommon in children, usually indicating *chronic pulmonary, hepatic, cardiac, or gastrointestinal disease*.

A 15-year old girl has only burning on urination and erythema surrounding the vaginal introitus. Her UA was negative for nitrites and leukocyte esterase. Likely diagnosis?

Chemical urethritis as a result of frequent douching, chemical irritants (soaps), fabrics (rayon), or drying agents (powders). Dysuria does not always indicate urethritis or UTI. Irritated UG lesions or chemical urethritis can promote dysuria with scant or no findings on physical or UA.

a. Cardiac sounds in a newborn: continuous machine-like murmur (PDA) in a premie with a widened pulse pressure, consider what infection? b. left-lower sternal border holosystolic murmur in a 3-month old? c. in a patient with Down syndrome? d. Fixed S2 splitting? e. Most common cardiac defect?

a. Congenital Rubella; b. VSD c. Endocardial cushion defect (both systolic and diastolic); d. ASD e.VSD

Explainable causes of sudden infant death can be divided into congenital and acquired conditions. What are they?

a. Congenital: cardiac anomalies (arrhythmia, congenital heart disease), metabolic disorders, and CNS etiologies. b. Acquired: infection and both accidental and intentional trauma.

In the first few months of life, if baby has staccato cough, conjunctivitis or mom chlamydia positive, consider:

a. Consider Chlamydia trachomatis. These babies will have eosinophilia and bilateral infiltrates with chest hyperinflation. b. Viral etiologies include herpes simplex virus (HSV), enterovirus, influenza, and RSV;

A 4-year old child complains of ear pain and malaise. He has a temperature of 102 and has had a cold for several days. Dx? Tx?

a. Dx = AOM (Acute Otitis Media) via penumatic otoscope b. Tx = oral antibiotics

In infectious mononucleosis, how is EBV contracted?

a. The EBV virus is excreted in saliva; infection results from mucosal contact with an infected individual or from contact with a contaminated fomite. a. Shedding of EBV in the saliva after an acute infx can continue for more than 6 mos, and occurs intermittently thereafter for life. Thus, EBV is ubiquitous in humans. Note: After an infection occurs, EBV replicates in the oropharyngeal epithelium and later in the B lymphocytes.

Physical findings to be considered in patients with nonspecific MR include?

a. size of the occiput b. unusual hair color or distribution, distinctive eyes, malformed ears or nose, and abnormalities in jaw size, mouth shape, or palate height. c. The hands and feet may have short metacarpals or metatarsals, overlapping or supernumerary digits, and abnormal creases or nails. c. The skin may have café au lait spots or depigmented nevi, and the genitalia may be abnormally sized or ambiguous.

Three days after beginning oral amoxicillin therapy for OM, a 4-year-old boy has continued fever,ear pain, and swelling with redness behind his ear. His ear lobe is *pushed superiorly and laterally*, Which of the following is the most appropriate course of action?

a.the child has mastoiditis- can require CT confirmation for Dx b.Treatment includes myringotomy, fluid culture, and parenteral antibiotics. Surgical drainage of the mastoid air cells may be needed if improvement is not seen in 24 to 48 hours.

WHat are the findings of nonphysiologic jaundice?

(1) appearance in the first 24 to 36 hours of life; (2) bilirubin rate of rise greater than 5 mg/dL/24 h; (3) bilirubin greater than 12 mg/dL in a full-term infant without other physiologic jaundice risk factors listed; (4) jaundice that persists after 10 to 14 days of life.

DKA tx pitfalls 2: Why is Bicarb infusions avoided?

(1) precipitate hypokalemia (2) shift the oxygen dissociation curve to the left, worsening organ oxygen delivery (3) overcorrect the acidosis, and (4) result in worsening cerebral acidosis while the plasma pH is being corrected (transfer into the cerebrum of CO2 formed when the bicarbonate is infused in an acid serum).

myotonic muscular dystrophy:

- AD - inverted V-shaped upper lip, thin cheeks, and wasting of the temporalis muscles - narrow head, palate is high and arched.

A 10-year old boy with multiple episodes of respiratory difficulty over the last 3 months presents with tachypnea, perioral cyanosis, accessory muscle breathing, slight wheezing and recent drowsiness. Mom gave him two nebulizer treatments that didn't help much. Dx? Next step?

- Asthma exacerbation. - Treat the patient's respiratory distress utilizing the ABCs of airway management. -Initial administration of oxygen, inhaled beta-agonist, and a systemic dose of prednisone is appropriate. Do blood gas, monitor O2 sat. - The noted 'drowsiness' indicates impending respiratory failure; further, note that wheezing may increase as therapy allows more air movement.

diff b/w TTP, ITP and TAR.

- TTP is a thrombotic microangiopathy more common in adults - ITP is an isolated thrombocytopenia, - TAR (Thrombocytopenia with absent radius (TAR) syndrome) is characterized by the absence of the radius in the forearm and thrombocytopenia

An 8-month old has refractory eczema and multiple episodes of AOM and PNA. He has now developed nosebleeds. His CBC reveals 15,000 platelets (thrombocytopenia). Diagnosis?

- Wiskott-Aldrich syndrome is an X-linked condition with recurrent infections, thrombocytopenia, and eczema. -Infections and bleeding usually are noted in the first 6 months of life. - Since there is T-cell dysfunction, pts will have poor antibody responses to capsular polysaccharides, fungal and viral septicemias. - In addition to eczema, these kids have autoimmune disorders and a high incidence of malignancy.

A mother brings a stool sample for your review. In the stool are several 15-to 20-cm long, round, whitish worms. You initiate treatment with which of the following?

- Worms of this size and description typically are Ascaris; - treatment is mebendazole or albendazole.

DMD symptoms

- asymptomatic during infancy with normal or mildly delayed developmental milestones - 3 -5 years have increasing lumbar lordosis (gluteal weakness), frequent falling, difficulty climbing stairs, hip waddle, and proximal muscle weakness (Gower sign). -Muscular enlargement, caused by hypertrophy of muscle fibers and infiltration of fat and collagen proliferation, causes calf, gluteal, and deltoid muscle pseudohypertrophyand a "woody" feel of the affected area. -Contractures of hip flexors, heel chords, and iliotibial bands develop, limiting joint range of motion. -Distal muscles remain functional, permitting manual dexterity. -Respiratory muscle involvement and the scoliosis result in diminished pulmonary function and recurrent pulmonary infections. -Oropharyngeal dysfunction can lead to aspiration, further compromising respiratory capacity.

juvenile pernicious anemia happens when?

- cant secrete IF - Vit b12 becomes deficient b/w age 1 and 2 yrs - maternal stores exhausted - irritability, loss of appetite, and decreased activity. - risk for perm neuro damage from spinal cord demyelinization. tx: IM vitamin b12 replacement.

Idiopathic thrombocytopenic purpura is

- common cause of bruising and petechia b/c of low plt levels - NO anemia, HSM..etc

What is intraventricular hemorrage?

- complication in preterm infants - a/w seizures, hydrocephalus, periventricular leukomalacia. - grade 4 bleed involves brain parenchyma - puts child at higher risk for neurodevelopment d.o

LAB data DKA:

- elevated glucose lvl (400-800 mg/dL) - metabolic acidosis (with anion gap) - hyperketonemia - hyponatremia -normal/elevated potassium (despite intracellular K+ depletion). - increased in BUN/Cr b/c of dehydration

Patau syndrome: Trisomy 13

- microcephaly -sloping forehead, - deafness - scal cutis aplasia - microphthalmia - coloboma - cardiac defect (VSD) -omphalocele - single umbilical artery - hypersensitive to atropine, pilocarpine

Cause of AD?

- multifactorial, involving genetic abnormalities of the epidermal barrier (mutation in filaggris FLG gene), immune function, environmental exposures, and infection

What happens in Tricuspid Atresia?

- no outlet b/w RA and RV = systemic venous blood enters LA via FO or ASD. VSD also often present. - ductal dependent

A father presents his otherwise healthy 15-month-old daughter to the emergency center with cough, post-tussive emesis, and subjective fever over the past 3 days. He also thinks her abdomen has been hurting her. Diarrhea started yesterday, with "regular" stooling prior to this illness. She has been drinking well and recently had a wet diaper. Physical examination reveals normal vital signs, congested nares, shoddy neck lymphadenopathy, and a mildly distended and apparently tender abdomen without obvious guarding. Which of the following is the next best step in your evaluation?

-Reassure parent and await spontaneous resolution. - Upper respiratory tract infection symptoms, neck lymphadenopathy, and diarrhea are consistent with viremia; - viral-mediated mesenteric lymph node enlargement can occur and cause nonspecific abdominal pain. - An abdominal CT scan may show diffuse, mildly enlarged lymph nodes in mesenteric lymphadenitis, but imaging is rarely warranted unless an etiology for abdominal pain remains elusive.

An unrestrained 2-month old infant is ejected through the windshield in a car crash. He has altered mental status, a bulging fontanelle with his skull sutures separated, bilateral retinal hemorrhages, and experienced a 2-minute seizure. What's the cause of his symptoms? How do you diagnose it?

-Subdural hematoma is likely and can be confirmed with a stat CT of the head--blood shows up white in a crescent moon-shape. - In a subdural bleed, blood from disrupted bridging veins, which connects cortex to dural sinuses, is found between the dura and the arachnoid space.

On physical examination of 2 y.o, you note lower extremity contractures, hand stiffness, somewhat coarse facial features, and hepatosplenomegaly, sister died d/t heart trouble. what is it?

-The enlarged liver + spleen, the coarse facies, and the history of a previous child's death from "heart trouble" point *to a storage disorder*. Her joint contractures and hand stiffness may be explained by an abnormal metabolism.

A mother notices a lump on her 5-year-old son's neck. He complains about pain in the region and difficulty swallowing. Appetite and intake are normal. On examination, he is afebrile with a 3-cm × 3-cm area of mild erythema, fluctuance, and tenderness of the central anterior neck. The mass moves superiorly when he opens his mouth. His oropharynx is clear. Which of the following symptoms was most likely present during the preceding week?

-Thyroglossal duct cysts, arising from the embryonic thyroglossal tract, are typically midline, often move on tongue protrusion, and often are noted after a URI. - Treatment is usually surgical excision, sometimes after neck CT imaging to ascertain cyst and thyroid anatomy. About half can become infected.

Neonates with GBS meningitis rarely have seizures as a presenting sign, yet 50% develop seizures within?

-within 24 hours of infection. - Major neurologic sequelae *(cortical blindness, spasticity, and global mental retardation) *occur in 12% to 30% of infants who survive meningitis

The parents of a previously healthy 3-year-old girl bring the child to your clinic because she is complaining that her tongue hurts. She is also weak and listless for mos and not eating well. Recently she has exhibited trouble walking. She eats regular diet, on exam examination, she is pale and tachycardic. Her complete blood count reveals a macrocytic anemia. You suspect a vitamin or mineral deficiency. What additional finding is most likely in this toddler?

. A smooth, red, and tender tongue may be observed in juvenile pernicious anemia, a rare autosomal recessive condition in which the child is not able to secrete intrinsic factor and cannot absorb vitamin B12.

How can you diagnose GERD? pH probe in the esophagus.

...

Girls with gonadotropin-independent, precocious pseudopuberty have?

....an independent source of estrogens causing their pubertal changes. b. An exogenous source of estrogen (birth control pills, hormone replacement) or an estrogen-producing tumor of the ovary or adrenal gland must be considered

Large VSDs are accompanied by

....by dyspnea, feeding difficulties, growth failure and profuse perspiration which all may lead heart failure (cardiomegaly on CXR). Note: Think about Greyson... FFT, on diuretics for CHF, and dyspneic--required surgery.

1) Most commonly used drugs in adolescence? 2) Most commonly used illicit drug in adolescence?

1) Cigarettes and alcohol; 2) marijuana

1) Bacterial meningitis in older kids is usually caused by _________ and ___________. 2) Bacterial meningitis in neonates/infants have non-specific symptoms and do not include the typical triad of headache, fever and stiff neck. Instead, infants exhibit thermal instability, poor feeding, seizures, and may have what cranial finding?

1) Streptococcus pneumoniae or Neisseria meningitidis. - Pneumococcal meningitis is MC in the winter, and since it is an encapsulated pathogen--children with poorly functioning/absent spleens (like sickle cell kids) are at much higher risk for infection. - Neisseria: usu d/t new infection rather than long term carriage, serotype B and C. 2) In addition to the other symptoms listed, infants may have a bulging fontanelle as well in bacterial meningitis.

CT VS MRI?

1. Cervical CT imaging is an excellent study for determining whether a patient has only cellulitis and edema surrounding a space, or hypodensity and rim enhancement consistent with an abscess. - It also delineates whether there has been extension to contiguous structures. 2. An MRI is an alternative when there is a concern for infection involving a compartment with neurovascular elements and more accurate visualization is desired.

First few DAYS of life: what are the bacterial etiologies?

1. Enterobacteriaceae and group B Streptococcus (GBS) = MC 2. others: s. aureus Strepto pneumoniae and Listeria. Tx. = ampicillin with either gentamicin or cefotaxime.

other problems with CF

1. Fatty liver infiltration or focal biliary cirrhosis 2. Hepatomegaly, esophageal varices, and hypersplenism caused by portal HTN 3. cholelithiasis in adults 4. neonates: blocked intrahepatic bile ducts = conjugated jaundice. 5. azoospermia, endocervicitis, enlarged salivary glands, and a "salty taste" on the skin

Adverse effects for antiretrovirals

1. HA 2. Emesis 3. abdominal pain 4. diarrhea 5. Osteopenia 6. drug rash 7. Anemia, neutropenia, elevated transaminases, hyperglycemia, hyperlipidemia.

An immature and insecure 13-year old male with hypogonadism, long limbs, gynecomastia, and difficulties in learning. Dx? Next step? Tx?

1. Klinefelter's Syndrome. Obtain chromosomal analysis; it is often 47,XXY caused by paternal > maternal nondysjunction. 2. NOTE: Consider this diagnosis for all boys (regardless of age) who have mental retardation or social/school problems. 3. Tx involves replacement therapy with a long-acting testosterone beginning at 11 or 12 years of age ideally.

The 3 major classes of antiretrovirals are?

1. Nucleoside reverse transcriptase inhibitors: didanosine, stavudine, zidovudine 2. Nonnucleoside reverse transcriptase inhibitors (enfavirenz, nevirapine) 3. protease inhibitors (indinavir, nelfinavir)

CF complications?

1. Pneumothorax, hemoptysis, and cor pulmonale. 2. pulmonary problems --> cause respiratory and cardiac failure. 3. Chronic nasal congestion and sinus opacification are common, but acute sinusitis occurs infrequently. 4. children may develop nasal polyps, with resultant nasal obstruction, headaches, and mouth breathing.

Clinical pearls

1. Vitamin B12 dietary deficiency is rare; infants breast-fed by vegan mothers are at risk to become vitamin B12 deficient and should receive supplementation. 2. Infants drinking goat's milk must be supplemented with vitamin B12, folate, and iron. 3. Vitamin B12 deficiency related to gastric antrum or ileal resection requires parenteral vitamin B12 supplementation. 4. Vitamin B12 deficiency can lead to permanent neurologic damage.

Klinefelter symptoms: A specific syndrome associated with

1. behavioral problems (immaturity, insecurity) 2. developmental delay (speech, language, lower IQ - especially verbal IQ) 3. physical findings (gynecomastia, hypogonadism, long limbs) caused by an extra X chromosome in boys and men.

SCD: A variety of screening tests, such as routine CBC and reticulocyte measurements, begin at?

2 months of age or at diagnosis.

Apnea is defined as "cessation of breathing for at least __ seconds that may be accompanied by bradycardia or cyanosis." Also, what does ALTE stand for?

20 seconds. Realize kids often have episodes of apnea and it is totally normal--usually they do not long 20 seconds and will grow out of this as their brain matures. ALTE stands for "apparent life-threatening event."

Prior chlamydial infection carries how much an increased risk for ectopic pregnancy in females? What is the treatment for GC and chlamydia?

2x increased risk. GC Tx = single IM dose of Rocephin (ceftriaxone) Chlamydia Tx = 1 dose of Azithromycin, or 1-week course of doxycycline. Realize that patients with GC infection are often treated for possible concomitant chlamydial infection.

Full-term newborns usually have peak bilirubin concentrations of what during the second and fourth days of life? Approximately what % of breast-fed full term infants develop significant unconjugated bilirubin elevations up to 30mg/dL after the 7th day of life?

5 to 6 mg/dL; Approximately 2%. Formula substitution for breast milk for 12-24 hours results in rapid bilirubin level decrease and breast-feeding can resume without return of jaundice.

At 1 minute, a newborn has a pink trunk but his extremities are blue, his HR is 120, showing some flexion of extremities, a facial grimace but not coughing, and is showing a good respiratory effort. What is the APGAR score?

7. APGAR stands for Appearance (color), Pulse (heart rate), Grimace (reflex irritability), Activity, and Respirations. Appearance--baby is all pink (2), trunk is pink but extremities blue (1). Pulse--above 100 (2), below 100 (1), absent (0). Grimace--cough or sneeze (2), grimace (1), no response (0). Activity (muscle tone)--flexed and moving (2), some flexion (1), limp (0). Respiratory effort--good, crying (2), slow and irregular (1).

Chromosomal analysis of DS shows?

95% Nondisjunction: failure to segregate during meiosis. 2% translocations 3% mosaics

A simple febrile seizures lasts for ______ minutes without focal or lateralizing signs. A complex febrile seizure lasts for ______ minutes and may have lateralizing signs. Realize that if a seizure lasts longer than 5 minutes, __________ can be used to interrupt the seizure. If that medication didn't work, you can try ____________.

< 15 minutes, > 15 minutes. Benzodiazepine (lorazepam or diazepam), fosphenytoin.

If you a newborn with stridor and subsequently has red, macular lesion that does not blanch upon pressing. What might you consider?

A capillary hemangioma around the subglottic area causing respiratory distress.

ACUTE OTITIS MEDIA (AOM):

A condition of otalgia (ear pain), fever, and other symptoms along with findings of a red, opaque, poorly moving, bulging tympanic membrane (TM).

What is the Somogyi phenomenon?

A. Nocturnal HYPOglycemic episodes manifested as *night terrors, headaches, or early morning sweating* and then presents a few hours later with *HYPERglycemia, ketonuria, and glucosuria*. B. Counter-regulatory hormones, in response to the hypoglycemia, cause the hyperglycemia.

What is Atopic Dermatitis? Contact Dermatitis? Eczema?

ATOPIC DERMATITIS (AD):A patch or plaque of erythematous skin with intense pruritus; the most common eczematous eruption in childhood. CONTACT DERMATITIS: An adverse reaction of the skin to an outside agent. Includes primary irritant dermatitis (eg, irritant diaper rash) and allergic contact dermatitis (eg, poison ivy, nickel allergy). ECZEMA: General term for a skin condition consisting of acutely inflamed papules and plaques, frequently a/w serous discharge and pruritus. Eczematous eruptions include atopic dermatitis, seborrheic dermatitis, and contact dermatitis.

In toddlers thru kids 5 years of age, viral pneumonia is common and will have interstitial markings on CXR. List some common viruses. What test can you do to confirm viral etiology?

Adenovirus, rhinovirus, RSV, influenza, and parainfluenza. PCR (nucleic acid amplification) of secretions from nasal swab can confirm viral etiology. Realize however about 20% of viral PNA have concomitant bacterial PNA!

If steroid production is the underlying etiology of the intersex problem, treatment is provided to prevent further virilization...what is the tx

Administration of hydrocortisone to individuals with CAH helps to inhibit excessive production of androgens and further virilization.

A 10-month old boy has a 1-day history of fever to 104, irritability, and decreased PO intake. Physical exam is normal, but the child is lethargic. Next step?

Although this child may be diagnosed with febrile seizures, he is lethargic and < 1 year of age so an LP is required to evaluate the child for meningitis.

MDMA ingestion in a toddler? symptoms =

An ingestion of MDMA can certainly cause seizure in a toddler but is usually associated with hypertension, dilated pupils, and hyperthermia.

KASAI PROCEDURE is?

An operative procedure in which a bowel loop forms a duct to allow bile to drain from a liver with biliary atresia.

How do Anticholinergics work?

Anticholinergics may be useful in the acute management but not chronic therapy; they work by inhibiting the vagal reflex at smooth muscles.

A 9-year old boy has 24 hours of persistent abdominal pain and vomiting. His physical reveals abdominal guarding and RLQ rebound tenderness. Diagnosis?

Appendicitis.

MECONIUM ASPIRATION SYNDROME is?

Aspiration of meconium during delivery resulting in respiratory distress. Radiographic findings include hyperinflation with patchy infiltrates. As meconium may plug small airways, areas of air trapping are often present and may lead to the development of pneumothorax.

A patient is suspected to have osteomyelitis. Since only half of these patients have positive blood cultures, what procedure should be done prior to antibiotic administration?

Aspiration of the bone. Staph aureus is the most common pathogen in all age groups! Patients with sickle cell disease are particularly susceptible to Salmonella osteomyelitis.

A 2-month-old girl with Down syndrome is noted to have: - a systolic and a diastolic heart murmur, and the second heart sound is split. Her mother reports that lately the baby has been sweaty and sometimes blue around the mouth when she nurses. Her electrocardiogram (ECG) shows a superiorly oriented QRS frontal plane axis with counterclockwise depolarization pattern and RVH. DX?

Atrioventricular canal defect

For an ADHD diagnosis to be made, symptoms must be present for at least __ months in two or more settings, with symptoms beginning before age ___. How do you treat ADHD patients?

Behavorial modification can be used alone or in conjunction with medications. First-line pharm therapy includes stimulants like methylphenidate and dextroamphetamine. Atomoxetine (Strattera) is a non-stimulant, SNRI that works as well. About 50% of children will function well in adulthood, while the others demonstrate continued inattention and impulsivity symptoms.

A newborn baby is jaundiced on the first day of life and has been passing whitish-clay colored stools since birth. Bilirubin levels were obtained, and came back as a high direct bilirubin. Diagnosis? Further, the new intern puts this baby under phototherapy causing the baby to have bronze skin, but the jaundice doesn't look much better. Dx? Reason?

Biliary atresia. Bronze baby syndrome. Photolight therapy does not make direct bilirubin soluble, therefore does not help excrete it and the therapy is useless. A Kasai procedure is necessary.

BRUDZINSKI SIGN? Kernig sign?

Brudzinski: A physical finding consistent with meningitis; while the patient is supine, the neck is passively flexed resulting in involuntary knee and hip flexion. KERNIG SIGN: A physical finding consistent with meningitis; while the patient is supine, the legs are flexed at the hip and knee at 90° angle resulting in pain with leg extension.

A mom brings in her 4-year old son who complains of right knee pain for 2 weeks, limps a little, is fatigued, and fever to 100.4. What's the most important lab test to get?

CBC with differential and platelets. This patient has symptoms of JRA and leukemia thus a CBC is the best initial screening test. JRA will have a normal CBC with no blast cells; ALL will have >5% blasts on peripheral blood smear.

A 7-month old is brought to the ER by paramedics as she experienced a seizure for 30 minutes. Three days ago her PCP diagnosed her with a URI. Vitals are HR 90, RR 25, temp 100.4. The tympanic membranes are dark and without normal landmarks, and the anterior fontanelle is full. Lungs are CTA bil, abdominal and heart exams are normal. She has a bruise over the occiput and along the spine. What test do you order?

CT of the head due to suspected intracranial hemorrhage from physical trauma.

A child with lymphadenopathy, organomegaly, weight loss, recurring infection and oral lesions is consistent with?

Candidiasis/immune system dysfunction

All DMD patients will have some degree of what heart issue? Patients generally become wheelchair bound by what age? NOTE: Nonprogressive intellectual impairment is common as patients have an average IQ of 80, and brain atrophy can be seen on CT.

Cardiomyopathy. Age 12.

What is a complication of DKA that can present with headache, personality change, vomiting, and decreased reflexes?

Cerebral edema.

SYSTEMIC-ONSET JIA:

Characterized by arthritis with: - fever - evanescent rash - hepatosplenomegaly - serositis - lymphadenopathy.

ASTHMA EXACERBATION:

Characterized by the triad of bronchoconstriction, airway inflammation, and mucus plugging.

Whenever you see epispadia or hypospadia, what is contraindicated?

Circumcision.

LGA infants are prone to trauma during delivery. What's the most common traumatic injury?

Clavicular fracture. Mom will say that she felt a knot/mass near the upper chest in the newborn, and you will also feel crepitus (feels like rice-krispies) during physical exam. Tx is not necessary and will heal on itself.

If a newborn still ends up being HIV+ after AZT prophylaxis was given for 6 weeks, what should the baby now receive at 6 weeks of age? What vaccines shouldn't these patients get?

Combination retroviral therapy (2 NRTI's + 1 protease inhibitor) and trimethoprim-sulfamethoxazole (for PCP prophylaxis). HIV+ patients should not receive live vaccines (influenza, MMR).

A 5 year old girl was playing with her twin brother when he accidentally poked her in the eye with a toy. She has been fussing lately saying her eye hurts now so mom brought her in. You notice over her iris, a white line. You suspect that she has a ________________ and can be diagnosed officially by examining the surface of the eye with what?

Corneal abrasion; It can be diagnosed officially by examining the surface of an eye by putting in fluoroscein drops and viewing under a blue-filter light. If a corneal abrasion is present, the abrasion will fluoresce.

STACCATO COUGH:

Coughing spells with quiet intervals, often heard in pertussis and chlamydial pneumonia

A 6-month old infant is FTT, even after his mom tried 3 different formulae. The child is pale, emaciated with little subQ fat and anterior fontanelle fullness. His lab results are notable for hemolytic anemia and prolonged bleeding times. What should you test for?

Cystic Fibrosis via a sweat test. This patient has deficiencies of vitamin K (prolong bleeding), vitamin A (anterior fontanelle fullness, inc ICP), and vitamin E (hemolytic anemia). Vitamins ADEK are fat-soluble.

TX: abs?

Depending on resistance: a. amoxicillin at doses up to 80 to 90 mg/kg/d for 7 to 10 days. b. Clinical failure after 3 days --> change to - amoxicillin-clavulanate - cefuroxime axetil - azithromycin - cefixime - ceftriaxone, or tympanocentesis is considered. c. Adjuvant therapies (analgesics or antipyretics) are often indicated, but other measures (antihistamines, decongestants, and corticosteroids) are ineffective.

A 2-hour newborn has micrognathia and just had a seizure. A CXR shows marked hypocalcemia, a boot-shaped heart and no apparent thymus. Dx?

DiGeorge Syndrome due to a 22q11 -microdeletion, chr 22 deletion. DiGeorge results from failure of the 3rd and 4th pharyngeal pouches to develop, thus no parathyroid and thyroid glands (thus hypocalcemia that can cause seizure, thymic agenesis, and no T-cell production). Patients have velocardiofacial defects such as VSD and Tetralogy of Fallot!

Parents worry about their 4-year old son's ability to walk. He first walked at 16 months but was clumsy and fell frequently. He remains clumsier than his peers, falls during simple tasks, and has developed a "waddling" gait. Within the last month he has experienced difficulty arising from a sitting position on the floor. Dx? Screening test? Diagnostic test of choice?

Duchenne Muscular dystrophy: - an X-linked recessive disorder is likely and is the MC hereditary neuromuscular degenerative disorder (Becker's is similar but less severe). - DNA blood analysis is diagnostic in 2/3 of cases. - Muscle tissue testing for abnormal dystrophin can be performed when blood samples are not diagnostic. - Muscle biopsy will show inflammatory, fat, and connective tissue infiltrates. - Other helpful lab findings include an elevated CK as an initial screen.

A 7-year old girl has itching for 1 week in her "private parts", abdominal pain, and a decreased appetite for a month. History excludes sexual abuse, contact dermatitis, and skin disorders. Exam is normal except mild perianal erythema. How do you make this diagnosis? Tx?

Dx = Cellophane tape on perianal area with microscopy will pick up Pinworm eggs (Enterobius vermicularis)--do this early in the morning. Tx = Mebendazole or pyrantel pamoate--note: family members who sleep in the child's bed should also be treated.

A term male is born vaginally to a 22-year-old primigravida woman; the pregnancy was uncomplicated. Just prior to delivery, fetal bradycardia was noted, and at delivery thick meconium is found. The infant has hypotonia and bradycardia. Which of the following is the first step in resuscitation?

Endotracheal intubation with direct suction is performed in a depressed infant with thick meconium noted at delivery. Bag-and-mask ventilation or endotracheal intubation without suction may increase the volume of meconium aspirated. A vigorous infant with a heart rate greater than 100 beats per minute, strong respirations, and good muscle tone with meconium-stained need not be suctioned immediately after birth.

If a patient has persistent torticollis, what facial abnormality can result?

Facial asymmetry can result if the condition persists beyond the first months of life; an ortho consult is indicated. Realize torticollis can result from birth trauma or vertebral malformation, or later in life from trauma, inflammation, neurologic conditions, or medications. Therefore torticollis presenting beyond infancy requires cautious evaluation.

Small left colon syndrome presents with?

Failure to pass meconium in the first 2 days of life with resultant abdominal distention and vomiting.

A 3-month old male infant has intermittent neck contortions and arching. He was term at birth with an uneventful prenatal course and delivery. He frequently spits up after feeding, and has had one episode of pneumonia. Diagnosis?

GERD + intermittent torticollis = Sandifer syndrome. He has a history of frequently spitting up and has had pneumonia, probably from aspiration. Sandifer syndrome patients have abnormal head posturing associated with reflux, probably to protect the airway.

If person ingests Elemental mercury in vapor form, what would be seen?

GI complaints would be seen, along with fever, chills, headaches, visual changes, cough, chest pain, and possibly pneumonitis and pulmonary edema.

Goats milk vs. cow's milk

GOAT = - lower sodium levels - but more potassium, chlorie, linoleic acid, and arachidonic acid - low in vitamin D, iron, folate, and vitamin B12

Gower Sign vs. TRENDELENBURG GAIT:

GOWER SIGN: A description of patients with proximal muscle weakness arising to a standing position. --> The legs are brought under the torso and weight is shifted to the hands and feet. The hands are walked toward the feet and up the thighs as the patient attempts to rise. TRENDELENBURG GAIT:A pelvic waddling gait from proximal muscle weakness.

A newborn has cataracts, jaundice, hypoglycemia, and vomiting.

Galactosemia. This condition is a contraindication to breastfeeding. Other contraindications include syphilis, varicella, active/untreated Tb, herpetic breast lesions, and AIDS/HIV.

GLOMERULONEPHRITIS is?

Glomerular inflammation resulting in the triad of hematuria, proteinuria, and hypertension.

What is Denys-Drash syndrome?

Group of findings composed of - Wilms tumor - gonadal dysgenesis, and nephropathy.

Is there is maternal history of IV drug use makes pediatric ___ infection a strong likelihood.

HIV infection = strong likelihood d/t vertical transmission.

What is Horner Syndrome?

HORNER SYNDROME: Characterized by eyelid ptosis and sluggish pupillary reflex; related to sympathetic nervous system dysfunction

How much nutrition (answer in kcal/kg/d) is appropriate for FTT and healthy newborns?

Healthy infants in the 1st year of life require ~120 kcal/kg/d of nutrition, and about 100 kcal/kg/d thereafter. FTT kids require an additional 50-100% to ensure good growth.

EBV was first identified from Burkitt lymphoma tumor cells. However, it is also associated with other malignancies such as what?

Hodgkin disease, nasopharyngeal carcinoma, and lymphoproliferative disorders. - EBV can stimulate hemophagocytic syndrome. - HIV-infected patients may develop oral hairy leukoplakia, smooth muscle tumors, and lymphoid interstitial pneumonitis with EBV infection.

If a posterior fontanelle happens to be open at birth, doesnt close by 6 months, and it is large, what condition are you thinking? Which one does NOT cross the suture lines: caput succundeaum or cephalohematoma? Which of the following is sharply demarcated: a bruise or a Mongolian spot?

Hypothyroidism. Cephalohematoma does NOT cross suture lines. Caput Succundeaum are the "conehead babies" and will resolve faster than cephalohematomas. Remember that the cephalohematoma is a risk factor for neonatal jaundice. Mongolian spots are sharply demarcated and are often near the sacral area.

The baby has a large posterior fontanelle, jaundice, and mottled skin. He doesn't pass stool very much, has an umbilical hernia, and a protruding tongue. Dx?

Hypothyroidism. T4 is low, and TSH is high.

CNS tumors are the most common solid tumors in pediatrics, and are second only to leukemia in overall incidence of malignancies. In contrast to adults, where 70% of tumors are supratentorial, 70% of child CNS tumors are _______________. The most common presenting symptom in kids with infratentorial tumors is _____________. The best imaging to screen is ________ and the gold standard for localizing brain tumors is __________.

Infratentorial (involving the cerebellum and brainstem). Childhood brain cancers are usually well-differentiated and are astrocytomas or medulloblastomas. Problems with balance or brainstem function (truncal ataxia, loss of coordination, wobbly gait, CN dysfx). It usually is the supratentorial tumors that cause signs of ICP and personality changes. CT of the head is used as an initial screen and reveals if there is hydrocephalus or hemorrhage. MRI is the gold standard for localizing brain tumors.

A 12-month old child drags his legs rather than using a four-limbed movement when he crawls. She says he only recently began crawling and cannot pull to a stand. At 6-months of age he hadn't yet rolled over nor sitting. On exam today, you note he positions his legs in a "scissoring" posture when held by the axillae. Initial step? Next step? Dx?

Initial step: Gather more detailed histories (pregnancy, birth, family, etc), especially developmental questions. Most likely diagnosis is Cerebral palsy. Next step: vision/hearing testing, consider brain MRI, and arrange session with developmental specialist.

PLEURAL RUB:

Inspiratory and expiratory "rubbing" or scratching breath sounds heard when inflamed visceral and parietal pleurae come together.

A 14-year old patient was visiting the coast with her family and had fish for dinner. Thereafter she developed a wobbly gait (ataxia), couldn't speak clearly (dysarthria), and had pins-and-needles sensation (paresthesia). Dx?

Methyl mercury intoxication from contaminated fish.

Breast feeding vegan mothers are given vitamin B12 to prevent the infant's developing?

Methylmalonic acidemia - (an amino acid metabolism disorder involving a defect in the conversion of methylmalonyl-coenzyme A to succinyl-CoA)

A HIV+ female is pregnant. How do you treat mom during the pregnancy and the newborn after delivery?

Mom should receive Zidovudine (AZT) at least by the second trimester and the newborn gets AZT through the first 6 weeks of life. This reduces transmission to < 10%. NOTE: HIV is typically acquired thru vertical transmission, but can still be passed on to baby at delivery and during breast-feeding thru infected secretions.

Hookworks:

Name: Ancylostoma duodenale and Necator Americanus Source: Larvae in soil --> skin Signs and Symptoms: pruritus and rash at site of entry, epigasric pain + diarrhea. Anemia, resp. symptoms. Dx: ovoid eggs in stool Tx: Albendazole, mebendazole, Pyrantel pamoate, Fe2+ supplements

Ascariasis:

Name: Ascaris lumbricoides Source: egg ingestion usually from soil contamination w. human feces Signs and Symptoms: asymptomatic - mostly, hemoptysis, pul infiltrates, abdominal pain, distention, obstruction. Dx: embryonate + non embryonate eggs in stool. Tx: Albendazole, mebendazole, Pyrantel pamoate, obstruction cleared w. piperazine salts.

Strongyloids

Name: Strongyloides stercoralis Source: larvae penetrate skin --> move to lungs and intestines. Also auto infectious, larvae can move from intestines --> blood stream, to lungs and back to intestines. Signs and Symptoms: asymptomatic or cause epigastiic pain, emesis, diarrhea, malabsorption, weight loss. Dx: larvae in feces or sample of duodenal fluid by a string test Tx: Ivermectin, thiabendazole

Two months after an episode of AOM, a 4 year old male still has middle ear effusion but isn't causing the child any hearing problems. Next step in management?

Observation is the usual response. However, if the fluid does not resolve or recurrent episodes of suppurative OM occur, myringotomy with PE tubes is often used.

By age 18, 1 in 4 women and 1 in 6 men are sexual abuse victims. What percentage of sexually abused kids have signs of physical trauma on examination? How many have victims have STDs?

Only 25% of these patients have signs of sexual abuse on physical exam, and even fewer have STDs. Realize that when injury does occur, it may be non-specific (i.e. erythema). Remember if the patient is a female of child-bearing age--always get a pregnancy test.

A 4-month old girl is fussy, appears to have pain on palpation of the right leg and has blue sclerae. Xrays reveal a right femur fracture. Her mom deny any severe trauma but reports that she had multiple fractures as a child. Dx? Family history will also include what other signs?

Osteogenesis Imperfecta is likely, an autosomal dominant disorder. Features include long bone fractures, deafness, short stature and blue sclerae. Specifically, this patient and her family probably have OI type 1 (mild). OI type 2 is lethal in utero, OI type 3 is most severe, and type 4 is moderately severe.

A 14y.o. had ataxia, euphoria, emotional lability, nystagmus and is abusive. Most likely drug?

PCP (Phencyclidine). Nystagmus and belligerence are hallmarks of PCP.

A 4-year old child is playing and becomes cyanotic; he then assumes a squatting position which relieves his symptoms. What defects characterize Tetralogy of Fallot? What does it look like on CXR?

PROV: Pulmonary stenosis, Right Ventricular Hypertrophy, Overriding aorta (overrides the ventricular septum), and VSD. ToF has a "boot-shaped heart" on CXR.

GESTATIONAL DIABETES:

Persistent hyperglycemia during pregnancy, with *serum glucose levels greater than 95 mg/dL in the fasting state* and above the thresholds for the oral glucose tolerance test.

It is in fall/winter and a 4 year old boy presents with fever, productive cough, and tachypnea. Coarse breath sounds are heard in all lung fields (rales) alongside bibasilar end-expiratory wheezes. Dx? Next step?

Pneumonia (=Lower resp tract infx) is a likely diagnosis that's made clinically + CXR. Thus, next step is CXR, pulse ox, and selected lab tests (CBC, cultures, nasal wash for antigens). ABCs are applied. Note a pulse ox is helpful because oxygen may be required in pts w/ PNA due to respiratory compromise (intubation is even used sometimes).

Fatty with satellite lesions on breasts, 2+ glucosuria, febrile. what is it?

Probably DM, do Hemoglobin A1c - cutatneous candidiasis = secondary immunosuppresion related to hyperglycemia. - in diabetics hyperglycemia promotes neutrophil dysfunction, and circulatory insufficiency contributes to ineffective neutrophil chemotaxis during infection.

A Caucasian term infant develops bilious vomiting and abdominal distention at 10 hours of age. No stool passages have been noted. AXR reveals distended bowel loops and a "bubbly" pattern in a portion of the intestine and the colon is narrow. Dx?

Probably more so cystic fibrosis than Hirschsprung's based on this x-ray. Meconium ileus, as seen here, is the earliest sign for CF and is a surgical emergency. It is nearly pathognomonic for CF.

EISENMENGER SYNDROME:

Pulmonary hypertension (HTN) resulting in right-to-left shunting of blood. This may occur with large ventricular septal defects (VSDs), atrioventricular canal lesions, and patent ductus arteriosus (PDA).

A 6-week old infant has projectile emesis after feeding. The vomit is non-bilious. He has an olive-shaped abdominal mass on abdominal exam. Diagnosis? What type of metabolic disorder will result? What procedure will confirm diagnosis?

Pyloric stenosis inducing a hypochloremic, hypokalemic metabolic alkalosis. The patient will be vomiting gastric contents which are high in HCl, thus you're losing chloride (hypochloremia) and losing H+ (metabolic alkalosis). Ultrasound is useful in confirming diagnosis. Pyloric stenosis usually affects male, first-born infants in the 3-8th weeks of life.

CAUDAL REGRESSION SYNDROME:

Rare congenital malformation found almost exclusively in the IDM, characterized by *hypoplasia of the sacrum and lower extremities.*

A 4-month old infant has excessive right-sided tearing. His mom says he gets irritable in bright light and calms down in a dark room. On exam, his right eye is larger than the left. What do you do?

Refer to a pediatric ophthalmologist. A history of 1) excessive tearing, 2) photophobia, and 3) corneal enlargement all suggest congenital glaucoma. Extra: Congenital rubella has a few eye complications including glaucoma, cataracts, and retinopathy.

An absent red reflex in an infant is a red flag for what cancer?

Retinoblastoma. the presence of leukocoria (absence of a red reflex, aka a "white" reflex) is a finding often seen with Rb and requires ophthalmogic referral for further work-up. It is Rb until proven otherwise. Realize though that cataracts (opacities of the lens) is the most common cause of leukocoria. Cataracts are often genetically determined, but may result from metabolic diseases or intrauterine infections (thus, congenital cataracts). If the cataracts are not surgically removed within 2 months, amblyopia (= reduced sight in the eye) results.

A 14-year old girl has a 3-day history of enlarged posterior auricular & sub-occipital lymph nodes with a diffuse salmon-colored rash. She reports a sore throat, cough, and low-grade fever for 5 days with tender swelling of multiple large and small joints. Dx?

Rubella. Know that vaccination is given at 1 year and again when the child is entering school. The major reason for vaccine is to prevent congenital rubella; the disease is mild in non-neonates.

PERITONSILLAR SPACE is?

SPACE: Bordered by tonsils and pharyngeal musculature; peritonsillar abscess is typically an extension of acute tonsillitis.

A 7-year old boy appears distracted. His mom notes that he daydreams "all of the time" and when he is daydreaming, he does not respond to her. She describes the episodes as lasting several seconds and occurs many times per day. When he is not daydreaming, he is attentive and completes tasks. Next step?

Send the child for an EEG as he probably has absence or petit mal seizures.

An 8-hour old newborn presents with bilateral conjuctivitis following routine newborn care in the nursery. Next step in the management?

Send the eye discharge for culture and start antibiotics based on culture results. Conjuctivitis in the first few hours of life is most likely caused by chemical irritation.

List the three main risk factors for drug use in adolescence.

Significant behavior problems, learning difficulties, and impaired family functioning.

What's the most common cause of septic arthritis? What should be considered in sexually active adolescents with septic arthritis? Sexually active adolescents should be tested for what? All sexually active women should get what at annual check-ups? What's the leading cause of death in adolescents?

Staph aureus; Neisseria gonorrhea; STDs (gonorrhea and chlamydia); pelvic exams; accidents/trauma.

AOM: Common pathogens seen in neonates + pts with immune deficiencies?

Staphylococcus aureus, Escherichia coli, Klebsiella pneumoniae, and Pseudomonas aeruginosa, are seen in neonates and patients with immune deficiencies.

Bacterial etiologies for neck abscess include?

Streptococcus pyogenes, Staphylococcus sp, Haemophilus influenzae, Peptostreptococcussp, Bacteroides sp, and Fusobacterium sp. Polymicrobial infection is typically seen, often reflective of the organisms most commonly found in infections involving the oropharynx, ear, or sinuses.

What is arthritis?

Swelling or effusion of a joint, or the presence of 2 or more of the following signs: - limited range of motion -tenderness or pain on motion - and increased heat in one or more joints.

A patient has exudative pharyngitis with neck LAN, rhinorrhea, and cough. Is it more likely to be viral or bacterial?

Symptoms are more consistent with viral etiology.

JRA is the most common rheumatologic disorder in kids. What age do you first have to have symptoms by, and how long do they need to last for a diagnosis of JRA? In the sexually active adult, what type of arthritis must always be considered.

The diagnosis specifies onset *prior to age 16* and *symptom duration 6 weeks or longer*. In the sexually active adult, gonococcal arthritis must always be considered. Septic joints should always be tapped.

A 5-year old girl has mild leg bowing and abnormal bone age for her ulna and distal radius. Calcium, phosphorus, AlkPhos, and urinary amino acid concentrations are all normal. Dx?

This is a classic presentation for Schmid metaphyseal dysplasia, an autosomal dominant disorder.

A 13-year old pubertal adolescent presents with pain in his right thigh. He said he had pain deep in the middle of this thigh a few weeks ago, but now the pain it is right above the knee. Pain and localized swelling are noted on physical exam. He has been fatigued lately and lost 10 lbs in 2 months. He has a fever today so his mom brought him in. Dx? X-ray reveals what?

This is most likely Ewing Sarcoma (t11;22), an undifferentiated bone sarcoma that arises primarily in adolescents that is 1.5x more common in males. In Ewing sarcoma, pain and localized swelling at the site of the primary tumor are the most common complaints. In the long bones, the pain usually begins midshaft, rather than at the ends as in osteosarcoma. Systemic manifestations are more common in children with metastases and include fever, weight loss, and fatigue. X-ray characteristically shows a lytic bone lesion with calcified periosteal elevation (onion skin) and a soft tissue mass. Bone marrow biopsy confirms diagnosis. Tx includes chemo with surgery or radiation; chemo is critical because it reduces tumor size/growth and treats microscopic metastases which are almost always present.

When should a baby be able to track and follow? sleep through the night? roll over? sit and drink juice?

Track and follow by 1 month of age, sleep through the night by 3 months of age, roll over at 4 months and sit/drink juice at 6 months.

A term 3500-g female delivered by cesarean section develops a respiratory rate of 70 breaths/min and expiratory grunting at 1 hour of life. She has good tone, good color, and a strong suck. Which of the following is the most appropriate next intervention?

Transient tachypnea of the newborn is a respiratory condition resulting from incomplete evacuation of fetal lung fluid in full-term infants - mc with c -sections, disappears w/i 24-28 hrs of life.

How do you treat eczema?

Treatment involves avoiding aggravating factors and drying soaps, and ensuring intensive skin hydration with emollients. - An emollient is a cream or lotion that restores water and lipids to the epidermis. - Creams are better than lotions because creams contain urea or lactic acid which make for a more effective lubrication. - Lower-potency steroids such as Fluticasone can be added to moisturizers to cover large areas of affected skin. - Fluorinated corticosteroids are avoided on the face, genitalia, and the intertriginous area because they may depigment and thin the skin.

A male neonate has known PUV and was sent for emergent vesicotomy because his urethral lumen was too narrow for a transurethral valve ablation. Following ablation and treatment, these patients still commonly have VUR, persistent hydronephrosis, and renal insufficiency. Because of these complications, what kind of things might you see in this kid?

VUR (vesicourethral reflux) is a common cause of UTI. Many patients will have polyuria because of diminished ability to concentrate urine (renal insufficiency) and are therefore at greater risk for dehydration.

A baby has hemihypertrophy, aniridia, and genital abnormality. What is he at risk for?

Wilms syndrome. This is classic WAGR syndrome.

A variety of pathologic conditions can cause nonphysiologic jaundice when excessive unconjugated bilirubin accumulates:

a. RBc lysing at rapid rate b.Transmission of unconjugated bilirubin to the liver is interrupted. c. Liver enzyme deficiencies --> no metabolism of unconjugated material.

Risk factors for neonatal physiologic jaundice?

a. maternal diabetes, b. cephalohematoma, c. male gender, Asian origin d. prematurity e. polycythemia f. trisomy 21 g. cutaneous bruising f. delayed bowel movement h. upper GI obstruction i. hypothyroidism j. swallowed maternal blood k. sibling w. physiologic jaundice.

Causes of nonphysiologic jaundice include:

a. septicemia b. biliary atresia c. hepatitis d. galactosemia e. hypothyroidism f. cystic fibrosis, g. congenital hemolytic anemia (eg, spherocytosis h. maternal Rh or blood type sensitization), i. drug-induced hemolytic anemia.

true precocious puberty stems from?

a. true precocious puberty stems from secretion of hypothalamic GnRH with normal-appearing, but early, progression of pubertal events. b. Sexual precocity is idiopathic in more than 90% of girls, whereas a structural CNS abnormality is present in 25% to 75% of boys.

Infants unable to digest protein require nitrogen in the form of?

amino acid mixture

Small VSDs are louder than large VSDs usually because

blood is pushing up against more resistance (a smaller hole); small VSD patients are asymptomatic.

In children with significant VSDs, chest radiography shows:

cardiomegaly and pulmonary vascular congestion, and the electrocardiogram (ECG) shows biventricular hypertrophy.

Polycythemia

central hematocrit >65.

Gingivostomatitis, tremor, and neuropsychiatric disturbances are seen with?

chronic inorganic mercury intoxication

Physiologic jaundice:

comprises primarily of *unconjugated hyperbilirubinemia* --> seen 1st week of life in approximately *60% of full-term infants and 80% of preterm* infants.

What are some sequelae can be seen? part 1

d. Further, with macrosomia, there is more of an oxygen demand so EPO is produced causing polycythemia, wwith resultant venous thromboses (kidney, sinus), stroke, necrotizing enterocolitis, or persistent pulmonary hypertension e. hypocalcemia is common and results in irritability, sweating, or seizures. e. IDMs are at increased risk for caudal regression syndrome, Congenital heart dis and Neural tube Defects.

Vitamin B12-mediated macrocytic anemia can occur as a result of?

dietary deficiency, malabsorption, or inborn errors of metabolism.

Western blot:

direct visualization of antibodies to Virion proteins, results can be indeterminate and require repeat testing.

EPIDURAL HEMORRHAGE: SUBDURAL HEMORRHAGE:

epidural: - Bleeding between the dura and the skull; - commonly occurs with skull fracture and middle meningeal artery laceration but can result from disruption of dural sinuses or middle meningeal veins. SUBDURAL HEMORRHAGE: - Bleeding between the dura and the arachnoid space -occurs with disruption of bridging veins connecting cerebral cortex and dural sinuses

Exposure to inorganic mercury salts (pesticides, disinfectants, explosives, dry batteries) can cause?

gastroesophageal burns, nausea, vomiting, abdominal pain, hematemesis, hematochezia, cardiovascular collapse, or death.

True contraindications to a particular vaccine include?

immediate hypersensitivity reactions to the given vaccine, the vaccine component, or the preservative in the agent.

Polyarticular disease:

is diagnosed when 5 or more joints are involved and systemic symptoms = mild or absent. a. mc in girls, biphasic age of onset (1-3y.o and 9-12 y.o) b. younger pts are RF -, older = RF + c. mc affected joints = shoulder, hips, temporomandibular, c-spine.

The amino acid-based formulas are excellent for children with?

malabsorption syndromes?

Low-isoleucine, low-leucine, low-valine infant formulas are useful for patients with?

maple syrup urine disease (MSUD);

Gilbert syndrome would present with a?

negative Coombs test, a normal (or low) hemoglobin, a normal (or slightly elevated) reticulocyte count, and prolonged hyperbilirubinemia.f

Boys with Klinefelter syndrome often go unidentified until puberty because?

of the subtleness of the clinical findings. The diagnosis should be considered for all boys (regardless of age) who have been identified as having mental retardation, or psychosocial, school, or adjustment problems.

if CP is suspected at birth what can be seen on imaging?

periventricular leukomalacia, atrophy, or focal infarctions.

Giving MMR or varicella to an asymptomatic patient with human immunodeficiency virus (HIV) is ____.

permitted

What happens in Liver failure?

poor bile salt secretion --> poor fat soluble vitamin absorption (incl vit d) --> low serum 25(OH)D, occasionally reduced serum calcium levels, HIGH serum alkaline phosphatase, poor bone mineralization, and an increased risk of fractures.

Atrial septal defects are well tolerated during childhood but can lead to?

pulmonary HTN in adulthood or atrial arrhythmias from atrial enlargement.

When monitoring children with large VSDs, one should not be misled by a softening murmur, as this may herald?

pulmonary vascular disease or infundibular stenosis rather than closure of the defect

Full-term newborns usually have peak bilirubin concentrations of 5 to 6 mg/dL between what days?

second and fourth days of life.

MYRINGOTOMY AND PLACEMENT OF PRESSURE EQUALIZATION TUBES:

surgical procedure involving TM incision and *placement of pressure equalization (PE) tubes (tiny plastic or metal tubes anchored into the TM)* to ventilate the middle ear and help prevent reaccumulation of middle ear fluid

Sepsis in the neonate can present with nonspecific findings of:

temperature instability, tachypnea, poor feeding, bradycardia, hypotension, and hypoglycemia.

Commonly found in such pesticides as parathion, malathion, and diazinon, organophosphates bind irreversibly to?

the cholinesterase of neurons and erythrocytes, as well as to liver pseudocholinesterase

HSV - intrauterine infection typically will cause ?

will cause chorioretinitis and microcephaly prior to birth.

Pharmacotherapyfor asthma includes

β-adrenergic agonists, anticholinergics, anti-inflammatory agents, and leukotriene modifiers.

CP -Concerning findings beyond infancy are what?

• A stepping response after the age of 3 months • A Moro reflex beyond 6 months • An asymmetrical tonic neck reflex beyond 6 months

What are the causes of Mental Retardation?

*1. preconceptual and early embryonic disruptions* a. teratogens b. chromosomal abnormalities c. placental dysfunction d. congenital central nervous system [CNS] malformations *2. fetal brain insults* a. infections b. toxins c. placental problems *3. perinatal difficulties* a. prematurity b. metabolic disorders c. infections *4. postnatal brain injuries* (infections, trauma, metabolic disorders, toxins, poor nutrition) *5. miscellaneous postnatal family difficulties* (poverty, poor caregiver-child interaction, parental mental illness).

Cough and increased respiratory effort in an afebrile infant with eye discharge are consistent with?

- Chlamydia. - 1/4th of infants born to moms w. chlamydia get conjunctivitis - Inner eyelid swabs are sent for PCR, and oral erythromycin or sulfisoxazole (latter only in infants older than 2 months of age) is given for 2 weeks for either conjunctivitis or pneumonia.

PARAPHARYNGEAL (LATERAL) SPACE is?

- Comprises anterior and posterior compartments containing lymph nodes, cranial nerves, and carotid sheaths. - infections in the lateral space can originate from the oropharynx, middle ears, and teeth.

Common morbidities of Cerebral palsy are?

- Deafness - visual impairments - swallowing difficulty with concomitant aspiration - limb and sensory impairments - behavioral disturbances

Apgar scores 0

- HR: Absent - RR: Absent - Muscle tone: Limp - Reflex irritability: no response - Color: blue, pale

______ ____ is generally indicated in delivering women with an outbreak of genital herpes or symptoms of HSV infection. If infection is primary, is infant risk higher or loweR?

- Cesarean delivery. - The infant's risk of HSV infection is increased significantly if the maternal outbreak represents primary infection.

What is Female pseudohermaphroditism?

- 46,XX karyotype - predominant etiology is CAH - other causes: a. exposure to maternal androgens/progestins b. congenital vaginal absence with uterine absence or abnormality; - degree of masculinization depends on stage of development at time of androgenic stimulation and potency and duration of exposure.

Children younger than 12 months at the time of their first seizure have a ___- __ chance of having another febrile seizure; older children have a 20% to 30% chance of recurrence. Children at highest risk for developing epilepsy following a febrile seizure often have?

- 50% to 65% - Children at highest risk for developing epilepsy following a febrile seizure often have.... preexisting neurologic problems and have complex febrile seizures; these children have 30 to 50 times the baseline risk of developing epilepsy

Symptoms of salmonella:

- Children usually have sudden onset of nausea, emesis, cramping abdominal pain, and watery or bloody diarrhea. - Most develop a low-grade fever - some have neurologic symptoms (confusion, headache, drowsiness, and seizures). - Between 1% and 5% develop transient bacteremia, with subsequent extraintestinal infections (osteomyelitis, pneumonia, meningitis, and arthritis)-- MC in IC and infants.

A newborn has contractures of multiple joints, facial wasting, generalized hypotonia, and weakness. The infant is transferred via helicopter to your facility. In your neonatal intensive care unit the infant's suck is noted to be weak suggesting that gavage feeds will be required, but the child's respiratory status worsens resulting in his requiring intubation and ventilator support. The mother of the child attends special education classes and walks with braces; Which of the following is the likely explanation for this child's condition

- A severe, congenital form of myotonic dystrophy can be seen in infants born to mothers with myotonic dystrophy. - Those that require ventilation longer than about 30 days have a poorer outcome.

SCID

- AR or X linked - both humoral and cellular immunity - serum IG and T cells = often diminished or absent - thymic dysgenesis - recurring cutaneous, GI, pulmonary infx occurs with oppurtunistic organisms such as CMV, PCP. - death in first 12-24 months unless BM transplant.

Volvulus? Radiographic findings:

- Abdominal radiographs may be normal or have nonspecific findings in cases of volvulus; thus an upper GI contrast series is generally indicated. - The characteristic finding in cases of volvulus is a "corkscrew" pattern of the duodenum or "bird's beak" of the second or third portions of the duodenum. - In cases of malrotation with or without volvulus, abnormal position (right sided) of the ligament of Treitz or malposition of the colon may be noted with contrast radiography.

True hermaphroditism:

- About 70% 46,XX and remainder 46,XY or mosaic - bilateral ovotestes or ovary and testis on opposite sides - testicular tissue determines virilization degree - gender assignment based on genitalia appearance (75% male)

A mother states that her 4-year-old son has had 2 days of "buttocks pain." She reports several bloodstreaked stools and frequent scratching of the area. He is afebrile, but his perianal region is bright red with a clearly demarcated erythematous border. The area is diffusely tender, but no nodularity, fluctuance, or trauma is found. Appropriate diagnostic testing and therapy include which of the following?

- Although diagnostic considerations should include pinworm infestation (as well as sexual abuse, contact diaper rash, and candidal diaper rash), the presentation is more consistent with perianal cellulitis. - Pinworm infection usually does not cause blood-streaked stool, and any erythema associated with it is not well demarcated. - Perianal cellulitis is commonly caused by Streptococcus and usually responds to oral or topical (mupirocin [Bactroban]) antibiotics.

Symptoms of ALL:

- Anorexia - irritability - lethargy - pallor - bleeding, petechiae - leg and joint pain - fever -hepatosplenomegaly (HSM)

What are some symptoms of lead poisoning?

- Anorexia, - Hyper-irritability, - altered sleep pattern, and decreased play are commonly seen. - Developmental regression, especially with speech - abdominal pain -Persistent vomiting, ataxia, altered consciousness, coma, and seizures are signs of encephalopathy. - long-term consequences= learning and cognitive deficits and aggressive behavior.

A 15-year-old adolescent female with periumbilical pain of 8-hour duration, followed by anorexia, emesis, and a loose bowel movement. She has no dysuria or sexual activity, and the pain appears unrelated to her menses. Her physical examination shows a quiet, rigid, tender abdomen, and pain with digital rectal examination. She has abdominal guarding and positive Psoas and Obturator signs. Most likely dx? Next step?

- Appendicitis is likely. -the initial abdominal pain followed by anorexia and vomiting suggests appendicitis. - pain = classically begins periumbilically and then migrates to the RLQ (McBurney's point). - pain can be laterally (retrocecal appendix), or it can be more diffuse (perforated appendix with resultant generalized peritonitis). -Psoas shadow obliteration, RLQ intestinal dilatation, scoliosis toward the affected region, and an appendicolith (seen in 10% of cases) support appendicitis. - A positive Psoas sign: Irritation of the psoas muscle caused by active right thigh flexion or passive right hip extension. -ROVSING'S SIGN: Palpation of the LLQ causes pain at the RUQ in patients with appendicitis. - A positive Obturator sign is elicited when you get pain by passive internal hip rotation. - Definitive tx is surgical removal of the appendix, surgery consult. - Abdominal US has great sensitivity, but many places use abdominal CT***. - A UA, CBC, and pregnancy test should be obtained.

A mother brings her 11-month old daughter into the clinic for a persistent facial rash. She is otherwise healthy. PE reveals dry, red, scaly areas on the cheeks/chin/peri-oral areas. The diaper area is spared. Dx? Next step?

- Atopic dermatitis (Eczema). - Next step is to obtain further history of rash duration and exacerbating factors, and family history for eczema/allergies/asthma. -Atopic dermatitis is pruritic, recurrent, and flexural in older kids and symmetrical in adults. - 65% of pts develops symptoms in the first year of life, while 90% will be diagnosed by age 5.

A 3-year old is at summer daycare and develops fever, cramping abdominal pain, watery diarrhea which progresses to small bloody stools, and is ill appearing. DDx? Summary: This child was exposed in his day care center and at home to gastrointestinal (GI) illnesses. He has fever, abdominal pain, and watery diarrhea that progressed to bloody diarrhea with mucus. He had a new-onset seizure.

- Bacterial enteritis with neurologic manifestations - Diagnostic tools: Fecal leukocytes, fecal blood, and stool culture. - Gastroenteritis caused by probably Shigella or Salmonella. - salmonella infections = self limited, tx only < 3 mos or IC pts - shigella: Tx to shorten the illness + decreased organism excretion. - Both are non-lactose fermenting, facultative anaerobe GNRs. Both are common in warmer months but with different routes of transmission. - Salmonella outbreaks occur sporadically but can be food-related as exposure to chicken and raw eggs are the most common source of infection. - Shigella is very infectious, is usually transmitted person-to-person especially in daycares. --> person to person, may occur thru food and water.

diagnosis of febrile seizure must be made only after considering?

- CNS infection as the cause. - 2 classic findings suggest meningeal irritation: Kernig and Brudzinski - If the neuro exam is abnormal after the seizure, the seizure occurred several days into the illness, or if the child is unable to provide adequate feedback during a neck examination, a lumbar puncture (LP) may be necessary. - do LP < 1 y.o with fever + seizure as meningeal signs not reliable

A 2-year old girl with a history of esophageal atresia and a VSD is hospitalized with Pneumocystis carinii pneumonia. This patient also had a history of hypocalcemia-induced seizure at age 18 months. Diagnosis?

- DiGeorge Syndrome--thymic hypoplasia results from abdnormal 3rd and 4th pharyngeal pouch formation during fetal development. - Neighboring structures formed during the same fetal growth period are often affected, thus some conditions include anomalies of the great vessels, esophageal atresia, bifid uvula, mandibular hypoplasia, and low-set notched ears.

Viral etiologies for neck abscess include?

- EBV, cytomegalovirus, adenovirus, and rhinovirus and may present similarly to bacterial infection. - Viruses can present with oropharyngeal exudate and swelling or neck masses in the form of lymphadenopathy.

Tx of early onset disease/infection? late onset?

- Early onset: combination of IV aminoglycosides (gentamicin or tobramycin) and penicillin (often ampicillin). - Late onset: of β-lactamase- resistant antibiotics (such as vancomycin) and second-or third-generation cephalosporins. - for infants with + cultures: tx for 10-21 days.

Causes of neonatal meningitis:

- Escherichia coli and group B Streptococcus. -Listeria monocytogenes and other organisms (Citrobacter sp, Staphylococcus sp, group D streptococci, and Candida sp) are less common - risk factors: LBW, preterm, moms with chorioamnionitis, prolonged rupture of membranes, traumatic delivery. - clinical symptoms: non specific, (often hypothermia), poor feeding, emesis, seizures, irritability, and apnea. Infants may have a bulging fontanelle, and they demonstrate generalized hyper-or hypotonicity. tx: neonate: ampicillin + third-generation cephalosporin or an aminoglycoside to cover infections caused by group B Streptococcus, L monocytogenes, and E coli. In suspected pneumococcal meningitis, a 3rd-gencephalosporin combined with vancomycin is often recommended.

A term infant is born vaginally following an uncomplicated pregnancy. Shortly after birth, he begins to cough, followed by a choking episode, difficulty handling secretions, and cyanosis. During the resuscitation, placement of an OG tube met resistance at 15cm. He is transferred to the NICU for management. Dx? Best test for evaluation? What pregnancy complication is often seen with this condition?

- Esophageal atresia, most likely a tracheoesophageal fistula. - Infants with 'esophageal atresia' cannot handle oral secretions and require constant esophageal pouch drainage to prevent aspiration. - A chest and abdomen x-ray with the OG tube in place will demonstrate a coiled tube in the esophageal blind pouch. - Polyhydramnios, because the fetus cannot swallow the amniotic fluid.

A 33-week old premie begins have respiratory problems soon after birth. He has poor oxygenation, grunting, and retracting. CXR reveals a reticulonodular "ground glass" pattern and decreased lung aeration. Dx? Tx?

- Evaluation of neonates born with respiratory distress and unilateral breath sounds includes an abdominal examination. - With asymmetrical breath sounds, pneumothorax and CDH are considered. -This infant's scaphoid abdomen suggests CDH; needle thoracostomy is avoided because intestinal perforation may occur. -The patient is stabilized and the need for ECMO is ascertained after the infant's initial therapy response is evaluated. Many cases of CDH are diagnosed by prenatal ultrasound.

Surgical intervention:

- Exploratory laparotomy is performed and bowel viability assessed. - Areas of necrotic bowel are removed and Ladd procedure of disengaging bowel with anomalous fixation and appendectomy are performed. - Complications = short gut syndrome if a significant portion of necrotic bowel is removed, and adhesions may develop leading to obstruction. - b.c of the significant mortality and morbidity a/w volvulus, asymptomatic patients with malrotation require surgical intervention.

Nonphysiologic etiologies are commonly diagnosed in a jaundiced infant who has?

- Fam Hx of hemolytic disease or - in an infant with concomitant pallor, hepatomegaly, splenomegaly, failure of phototherapy to lower bilirubin, vomiting, lethargy, poor feeding, excessive weight loss, apnea, or bradycardia.

An otherwise healthy 2-year old boy with normal development and a family history of seizure in his father (1 episode at age 4), has a brief generalized seizure that lasted 5 minutes. When the ambulance arrived, vitals were HR 108, RR 16, BP 90/60, and temp to 104. Dx? Management? Expected course of this condition? Summary: An otherwise normal 2-year-old boy, with a family history of a single seizure in his father at 4 years of age, has a brief, generalized, self-limited seizure associated with an elevated temperature. His examination is nonfocal. He has completely recovered within 1 to 2 hours of the seizure.

- Febrile seizures are common, occurring in 2-4% of kids; they seem to have a genetic basis and kids are at increased risk -if a first-degree relative has been diagnosed with them. - Best management is parental education, injury prevention during the seizures, and fever control. - Expected course includes possibility of more seizures if fever present, but he is likely to "grow out" of the condition by age 6. - He is likely to have no long-term sequelae and will have normal development.

16-year-old adolescent girl who has been seen several times per week over the last 2 months complaining of cough, occasional hemoptysis, malaise, and intermittent low-grade fever. Thus far you have identified a microcytic, hypochromic anemia for which she has been taking iron (without response) and migratory patchy infiltrates on chest radiograph that seem unaffected by antibiotic treatment. She has no tuberculosis (TB) exposure risks, and her TB skin test was negative. Today she also complains of facial edema and tea-colored urine. You suddenly realize her symptoms can be grouped as which of the following syndromes?

- Goodpasture syndrome is the clinical diagnosis when patients exhibit nephritis and pulmonary hemorrhage. - It can be caused by SLE and HSP.

An infant with a history of recurrent pneumonia is diagnosed with TEF at 8 months of age. The infant most likely has what type of TEF?

- H-type. H-type TEF infants are found later in infancy with recurrent PNAs and/or difficulty feeding. - Contrastingly, patients with esophageal atresia and a distal fistula will present in the first hours of life. Nonetheless, all patients with TEF are at high risk for GERD.

Apgar scores 1

- HR: <100 bpm - RR: Slow, irregular - Muscle tone: some flexion of extremities - Reflex irritability: Grimace - Color: Body pink, acrocyanosis (extremities blue)

Apgar scores 2

- HR: > 100bpm - RR: good crying - Muscle tone: flexed, active motion - Reflex irritability: cough or sneeze - Color: pink

A female college student has 10 days of malaise, headache, and nausea. She now has a fever, sore throat, and morbilliform rash after taking ampicillin. Her examination reveals a fever, rash, tonsillar hypertrophy with exudate, posterior cervical lymphadenopathy, and splenomegaly. She has an elevated WBC count with a lymphocytic predominance, and a mild thrombocytopenia. She has a WBC count that's lymphocyte-dominant. Dx? Best tool to diagnose? Management? Expected course?

- Infectious mononucleosis due to EBV infection. - Best tool to quickly confirm diagnosis is a Heterophil antibody assay (Monospot)--this is only reliable in kids older than 5! If the patient is < 5y.o., order specific antibody assays. - Symptomatic care should be given with avoidance of contact sports for a few months because if the spleen ruptures, blood loss can be fatal. -The acute illness should be over in 2-4 wks with gradual recovery. Although patient will deny recent ill contacts, EBV infection has a 30-50 day incubation.

A 6-week old baby has his umbilical cord still attached and shows evidence of delayed wound healing. Dx?

- LAD (Leukocyte adhesion deficiency). Marked neutrophilia is present because WBCs are unable to leave the blood.

What is LAD

- LAD: inheritable disorder of leukocyte chemotaxis and adherence by recurring sinopulmonary, oropharyngeal, cutaneous infections with delayed wound healing. - severe infx possible with staph, enterobact, candida

What is the term defined as "excessive and/or prolonged uterine bleeding with a regular menstrual cycle"? What is the term defined as "irregular uterine bleeding between menstrual cycles"?

- Menorrhagia; metrorrhagia. - Both are terms that fall under dysfunctional uterine bleeding. - The typical presentation is that of a teen with regular menstrual cycles that then develops prolonged/heavy bleeding or irregular bleeding.

Mixed gonadal dysgenesis:

- Most 46,XY/45, XO karyotype - testis with Sertoli and Leydig cells, but no germinal elements, on one side and streak gonad on other; - hypospadias, partial labioscrotal fusion, and undescended testes most common (incompletely virilized male appearance); - usually assigned Female gender and undergo gonadectomy (25% of streak gonads develop malignancy) - assign as male if testes descended.

A 10-day old previously healthy infant with a 12 hour history of fever, irritability, and decreased oral intake. Mom did receive prenatal care. The child has multiple 2mm fluid-filled vesicles on the parietal scalp and experienced a 2-minute, right-sided body shaking seizure. Laboratory studies reveal lymphocytic meningitis and thrombocytopenia. Dx? Work-up?

- Neonatal herpes. - A young infant with fever and irritability is presumed to have a serious bacterial/viral infection. - Despite mom having prenatal care, its possible she never had an outbreak to know that she has herpes. -In addition to CBC, work-up includes HSV cultures (blood, nasopharynx, eyes, urine, rectum, CSF). CSF is tested by PCR for HSV. -Left untreated, the majority of infants with disseminated or CNS infection die. - The use of high dose, long duration Acyclovir has reduced mortality and morbidity. - HSV encephalitis tends to be global in the neonate, but EEG/MRI in older patients show HSV has tropism for the temporal lobe.

A father reports his 3-year old daughter has decreased energy, loss of appetite, and an enlarging abdomen over the past few weeks. PE reveals discoloration underneath the eyes, proptosis, and a large, irregular abdominal mass along her left flank that crosses the midline. Dx? Next step? Tx?

- Neuroblastoma = a tumor of primitive neuroendocrine tissue (PNET tumor) usually from the adrenal gland. -This child has classic ecchymycosis (raccoon eyes), proptosis (sluggish sympathetics), and a non-tender abdominal mass that crosses the midline. -Also prevalent in neuroblastoma is opsoclonus-myoclonus syndrome, characterized by chaotic eye movements and myoclonic jerks (dancing eyes, dancing feet). -Next step is to get CT or MRI to assess the extent of the tumor. -Lab markers include elevated urinary vanillylmandelic acid and homovanillic acid levels (NE and dopamine metabolites respectively). Tx = surgical tumor excision + chemo/radiation

A 6-year-old boy who recently moved from the southeastern United States complains of "something coming out" of his buttocks while straining during defecation; it seems to resolve when he relaxes. He also complains of abdominal pain and bloody stools for the last week. Examination reveals a normal external anus without evidence of trauma. When straining, he produces a pink mucosal mass from his anus; it returns when he relaxes. Initial diagnostic evaluation should include which of the following studies?

- Pinworms are not known to cause rectal prolapse, but whipworms (Trichuris trichiura) are. - The whipworm nematode lives in warm and humid areas and is commonly found in the rural southeastern United States. - Routine microscopy for ova is sufficient for the diagnosis (whipworms produce many more ova than do pinworms). - Treatment is albendazole or mebendazole. - Cystic fibrosis should be a consideration in a child with rectal prolapse, although the history should also include frequent pneumonias, failure to thrive, or foul-smelling stools

An 8-year-old girl arrives at your clinic complaining about a minimally itchy rash on her chest, abdomen, and arms. It started with one small, scaly, red area on her chest and then spread. She is taking no medications. Physical examination reveals salmon-colored, flat, finely scaly, oval eruptions on her chest, abdomen, back, and upper arms. Which of the following is appropriate advice or therapy?

- Pityriasis rosea is preceded by a "herald patch," an annular, scaly, erythematous lesion. - The lesions are salmon-colored and in a Christmas-tree formation, following the lines of the skin. - The cause is unknown. -treatment may include antihistamines, topical antipruritic lotions and creams, low-dose topical corticosteroids, and phototherapy. -The rash usually lasts up to 6 weeks and then resolves. - It can be confused with nummular eczema and tinea versicolor. In the sexually active adolescent, syphilis should also be considered.

You examine a full-term 3780-g newborn in the nursery and notice that he has marked hypotonia, a very small penis, and unilateral cryptorchidism. Most likely diagnosis?

- Prader-Willi is most likely as it is characterized by severe hypotonia, FTT and hypogonadism early on. - Around age 6, hyperphagia, obesity, and mental retardation manifest. - Unilateral cryptorchidism is a distractor in this vignette--it's reasonable to associate it with mixed gonadal dysgenesis.

4-year-old complains of sore throat and difficulty swallowing for 3 days. She has been irritable and does not want to move her neck. Her appetite and intake have decreased, and she has vomited twice overnight. She exhibits no symptoms of (URI). She is otherwise healthy with up-to-date immunizations. Her PE is remarkable for fever to 102°F (38.9°C), bilateral tonsillar exudates, and an erythematous posterior oropharynx with right posterior pharyngeal wall swelling. Dx? Next step?

- Retropharyngeal abscess. - Lab test may include rapid strep test. - Imaging may include lateral cervical x-ray and CT//MRI to elucidate location and extent of infection. - The retropharyngeal space is behind the esophagus and contains lymphatics that drain the middle ears, sinuses, and nasopharynx. Because it's a space contiguous with the posterior mediastinum, there's a potential for the infection to spread. A patient who passively refuses to move the neck secondary to pain is likely to have a retropharyngeal abscess. A typical pt with retropharyngeal abscess is a toddler younger than 4.

A mother brings her 2-week-old son to the clinic for a well-baby visit. Her only concern is a rash on his face and scalp that began a week earlier. Examination reveals a healthy white male with normal vital signs and a normal examination except for yellowish, waxy-appearing, adherent plaques on the scalp, forehead, cheeks, and nasolabial folds. Which of the following therapies is appropriate for this condition?

- Seborrheic dermatitis presents in infancy and adolescence. - The chronic, symmetrical eruption, characterized by overproduction of sebum, affects the scalp, forehead, retroauricular region, auditory meatus, eyebrows, cheeks, and nasolabial folds. - More commonly known as "cradle cap" in infants, this self-limited eruption typically develops between 2 to 3 months of age primarily on the scalp. -The scale is yellow and waxy, and typically comes off with frequent shampooing. - The scale may be loosened with a small amount of oil. - In infants who do not respond to shampooing with baby shampoo, an antidandruff shampoo containing antifungal medication (Nizoral) or selenium may help, as will low-to medium-potency topical corticosteroids

A 19-year old girl has a 101.2 temp, lower abdominal pain, and dyspareunia. She denies n/v/d and has cervical motion tenderness on exam. Urine pregnancy test is negative, as was the US for appendicitis. Appropriate outpatient management includes what?

- Since this patient likely has PID (Evidenced by lower abdominal pain, no obvious cause, and cervical motion/adnexal tenderness), she should be treated for both gonorrhea and chlamydia. - A single dose of ceftriaxone and doxycycline for 1 week (or z-pak) is appropriate.

A 14-year old adolescent male with HIV and AIDS presents for a physical before traveling to SE Asia. In counseling him, you mention that he must always wear shoes to prevent against Strongyloides infections--why?

- Strongyloides can develop a "hyperinfection" in immunocompromised hosts. - Its life cycle does not require a period outside the host and thus can auto-infect the host. - This autoinfection can lead to disseminated strongyloidiasis in immunocompromised people.

CONGENITAL DIAPHRAGMATIC HERNIA (CDH):

- The condition of herniation of abdominal contents through the posterolateral foramen of Bochdalek into the thoracic cavity. - The incidence is approximately 1 in 5000 live births.

What is Wilms tumor?

- These tumors typically are a/w hematuria, hypertension, and a localized abdominal mass that rarely crosses the midline. - In general, patients with neuroblastoma are slightly younger and sicker than patients with Wilms tumor.

After the infant discussed above is stabilized and admitted to the NICU, a chest radiograph reveals bilateral patchy infiltrates with coarse streaking and flattening of the diaphragm. He abruptly has an increased oxygen requirement. Physical examination reveals decreased right-sided breath sounds. Which of the following is an accurate statement?

- This infant likely has a right-sided pneumothorax; excessive light transmission by transillumination and right-sided hyperresonance with auscultation are expected. -Infants with meconium aspiration and respiratory distress are at higher risk for pneumothorax, especially if high PEEP is used for oxygenation. - A chest tube for the pneumothorax may be needed. - Infants with severe respiratory distress or circulatory involvement may require emergent needle aspiration

A 12-year-old healthy boy has noticed some muscle weakness. He has experienced increasing difficulty lifting his backpack and walking long distances. He has no trouble with schoolwork, and he continues to play the piano and video games without tiring. His 38-year-old maternal uncle recently became wheelchair-bound for unclear reasons. Which of the following is the most likely diagnosis?

- This patient does not have muscle weakness that precludes extended use of distal muscles (hands) or limits his manual dexterity. - The child's presentation at age 12 years and a 38-year-old, wheelchair-bound maternal uncle suggest a diagnosis of Becker MD.

A 9-year-old girl complains of sore throat and anterior neck pain of 1-day duration, and nasal congestion and cough over the past 3 days. There has been no nausea or change in appetite. She describes "lumps growing in her neck" over the past day. Her past medical history is unremarkable. She is afebrile with a clear posterior oropharynx and a supple neck. She has four firm, fixed, and minimally tender submandibular masses without overlying skin changes; the largest mass is 1 cm in diameter. Which of the following is the most likely explanation for these findings?

- This patient has viral URI symptoms, most likely causing reactive lymphadenopathy. - Supportive care such as analgesics would be a reasonable treatment recommendation. - Rapid streptococcal testing usually is not warranted for classic URI symptoms; - streptococcal pharyngitis more commonly presents with sore throat, headache, nausea, and/or fever. - Signs of viremia and her neck examination do not suggest sialadenitis or neck abscess.

A teenage boy complains of a several-week history of facial "zits" that are painful and itchy. There are no other breakouts. He has inflammatory papules and pustules in the beard and moustache area and has mild cervical lymphadenopathy. He occasionally works weekends on a farm. Which of the following therapies is appropriate?

- Tinea barbae is caused by various dermatophytes and closely resembles tinea capitis. It can be acquired through animal exposure and is more common in farmers. - Topical antifungal preparations are ineffective; oral antifungals are required.

A term male is born at 38 weeks via a scheduled repeat C-section prior to onset of labor. Mom received good prenatal care and was GBS negative. At delivery, amniotic fluid was clear and APGAR scores are 8 and 8 and 1 and 5 minutes, respectively. Within the first hour of birth, he has tachypnea, nasal flaring and mild contractions. Chest auscultation reveals good aeration bilaterally. Diagnosis? Best management for this condition?

- Transient tachypnea of the newborn. - Supportive care including supplemental oxygen if needed. - The first thing to do with this kid is to listen to his heart and lungs! CXR may show perihilar streaking and fluid in the fissures with well aerated lungs. - TTN is a self-limited condition (resolves in 1-2 days) caused by slow absorption of fetal lung fluid with resultant tachypnea. - The condition more commonly is associated with C-section, term infants.

Acute lymphoblastic leukemia affects?

- affects the lymphoid cell line - and comprises approximately 75% of leukemia cases in children. - peak incidence of 2-4 years, mc in boys - those with down syndrome and fanconi anemia = increased risk for ALL.

Acute myeloblastic leukemia (AML)

- affects the myeloid cell line (granulocytes, monocytes, and can affect erythrocytes or megakaryocytes) - and comprises approximately 20% of childhood leukemia.

Pityriasis rosea:

- benign, possible viral cause -The lesions of align along cutaneous cleavage planes resulting in a "Christmas tree" pattern. - first herald patch: Single round or oval lesion of about 1 to 10 cm in size with a raised border and a scaly appearance. - subsequent wide spread eruption occur in crops and are typically *less than 1 cm in size, are oval or round, have a raised edge, and are pink to brown in color.

Tests for HSV>?

- blood, urine, csf ---> bac cultures - HSV cultures obtained from the blood, nasopharynx, eyes, urine, stool or rectum, CSF, and from any vesicular lesion. - csf tested by PCr - Pending test results, this infant is placed on IV antibiotics and antiviral therapy. - viral culture of samples taken from various body sites and PCR of CSF are the most useful diagnostic tests - Tzanck prep + Ag detection methods = rapid dx, LOW sensitivity. -Infected individuals often have moderate peripheral leukocytosis, elevated serum liver transaminase levels, hyperbilirubinemia, and thrombocytopenia.

Infants born to mothers with poorly controlled gestational diabetes are at risk for?

- congenital heart anomalies - hypertrophic cardiomyopathy - septal hypertrophy - conotruncal anomalies - subaortic stenosis

Features of Pulmonary Valve stenosis?

- cyanotic + exercise tolerance = proportion to degree of stenosis -upper L sternal border SYSTOLIC mumur --> radiates to left infraclavicular area - systolic click - ecg: normal in mild cases, greater degree pf stenosis = RAD, RA + RV hypertrophy - tx = valvuloplasty - seen in glycogen storage disease and Noonan syndrome (short stature, facial dysmorphism, and a wide spectrum of congenital heart defects)

Digeorge summary:

- decreased T cell prod + recurring infection -velocardiofacial defects such as VSD and TOF - thymic or Parathyroid dysgenesis + hypocalcemia + seizures - older pts = developmental and speech delays.

What is DKA?

- deficient insulin availability---> leading to lipid oxidation and metabolism rather than glucose metabolism. - no insulin =results in free fatty acid (FFA) released from adipose tissue and in unregulated hepatic FFA oxidation and ketogenesis.

Analysis and tx:

- dna blood analysis = diagnostic - muscle biopsy testting = endomysial CT proliferation, inflammatory cell infiltrates, areas of regeneration interspersed with areas of degeneration, and areas of necrosis. - elevated CK Treatment: - therapies to slow disease progression. - Orthopedic intervention, including bracing and tendon lengthening, can prolong ambulation and slow the progression of scoliosis. - caution with surg - prone to hyperthermia with anesthesia. - Respiratory failure is often the cause of death - monitor nutritional status.

Causes of jaundice within first 24 hrs of life?

- erythroblastosis fetalis - hemorrhage - sepsis - cytomegalic inclusion disease -rubella -congenital toxoplasmosis

methylmalonic acidemia symptoms:

- failure to thrive - seizure - encephalopathy -stroke, or other neurologic manifestations.

Disadvantages of breast milk:

- hiv/viral transmission - jaundice exacerbation d/t increased unconjugated bilirubinemia levels - low vit k levels (can cause hemorrhagic dis of newborn)

Tx of neck abcesses?

- intravenous penicillins, advanced-generation cephalosporins, or carbapenems. - Clindamycin or metronidazole is added if anaerobes are suspected and broad coverage is desired. - Clindamycin often is a good choice for monotherapy in the patient with penicillin allergy. - Broad spectrum antibiotics are started in the patient with neck abscess, with treatment modification if an organism is identified from oropharyngeal or surgical samples.

Hemolytic Uremic syndrome:

- irritability - pallor - bloody diarrhea - anemia - thrombocytopenia - decreased urine output - HTN

Children with CF grow poorly because?

- maldigestion from exocrine pancreatic insufficiency --> which can lead to abdominal distention, rectal prolapse, minimal subcutaneous fat and muscle mass, and frequent passage of oily, malodorous, floating stools. - resulting fat soluble vitamin deficiences occur - glands become distended with secretions--> meconium illeus or intestinal obstruction can occur.

Classic symptoms of meningitis seen in older children + adults =

- mental status changes - nausea - vomiting - lethargy, restlessness, ataxia, back pain, Kernig and Brudzinski signs, and cranial nerve palsies. - 1/4 have seizure -Patients with N meningitidis can have a petechial or purpuric rash (purpura fulminans), which is a/w septicemia

Assessment: CAH

- obtain careful history - fam pedigree - investigate maternal exposure to androgens - thorough PE - critical finding = presence or absence of testis in labioscrotal compartment. - other physical findings = a. hyperpigmentation of the labioscrotal folds (common in infants with CAH) b. phallic size and location of urethral opening c. uterus on bimanual examination d. evidence of failure to thrive (failure to regain birth weight, progressive weight loss, vomiting) + dehydration. -b.c ambigious external genitalia = may reinforce doubt about sexual identity = reconstructive surgery done usu. before 6 months of age.

Henoch - Schonlein purpura

- purpuric lesions, esp on LE - joint pain - blood in stool (guaiac positive)

Nutritional rickets is?

- results from resulting from inadequate dietary vitamin D or a lack of sunlight exposure - rare in industrialized countries.

What is Tinea corporis?

- ringworm - superficial cutaneous fungal infection caused primarily by Microsporum canis, Trichophyton tonsurans, T rubrum,andT mentagrophytes -starts as an erythematous papule that expands to form a circular, scaly, and erythematous lesion with raised borders, as it gets larger = central clearing. tx: topical azoles (eg, ketoconazole, clotrimazole) or systemic antifungals (such as griseofulvin) in more diffuse cases.

signs and symptoms of TTN?

- self-limited - It is felt to be caused by slow absorption of fetal lung fluid. -Infants with TTN develop respiratory distress shortly after birth with tachypnea, mild retractions, nasal flaring, and in more severe cases grunting and cyanosis. -Chest radiography reveals perihilar streaking and fluid in the fissures; lungs are aerated. -Most infants with TTN have resolution of their tachypnea in 24 to 96 hours.

Isotretinoinis the treatment of choice for?

- severe, resistant nodulocystic acne. - A 5-month course often clears a severe case of acne. - It is highly teratogenic and has many side effects, including cheilitis, conjunctivitis, hyperlipidemia, blood dyscrasias, elevated liver enzymes, and photosensitivity. -Lipid levels, liver enzymes, and complete blood counts should be monitored monthly during the course.

Definitive TX of appendicitis?

- surgery - For perforated appendicitis, initial management consists of IV antibiotics and fluid replacement; its course may be complicated by sepsis, abscess formation, or prolonged (4-5 days) paralytic ileus. - Percutaneous drainage catheters can be used to drain the abscess and then appendectomy is performed at a later time.

Renal vein thrombosis can present as an abdominal mass because?

- the kidney becomes congested and palpable. - other symptoms = *gross hematuria and thrombocytopenia* are seen in 25% of patients, *oliguria*

Trisomy 18: edward's

- weak cry - single umbilical art. - micrognathia - small mouth and high arched palate - clenched hand with overlapping of index over 3rd finder - simian crease - rocker bottom feeties - small pelvis and short sternum (aakash)

Fx of Turners?

- webbed neck - high arched palate - increased nevi - renal anomalies - increased arm-carrying angle - edema of the hands and feet. - a height less than the 5th percentile - no signs of secondary sexual characteristics - a small mandible, low posterior hairline - prominent ears, and a broad chest. - Treatment includes recombinant human growth hormone and estrogen replacement therapy.

Wiskott-Aldrich syndrome (WAS)

- x- linked - recurrent bacterial infx - bleeding 2nd to bleeding secondary to thrombocytopenia (1000 - 80,000 platelets/mm3) - plt dysfunction - chronic dermatitis -Autoimmune hemolytic anemia occurs in about a third of these patients. - BM transplant

Symptoms of nonnutritional rickets is familial, primary hypophosphatemia?

-Children at the age of walking present with *smooth* lower extremity bowing (as compared to angular bowing of calcium-deficient rickets) -a waddling gait - genu varum, genu valgum - coxa vara -short stature. - intraglobular dentin deformities (whereas ca2+ def rickets causes enamel defects) - radiologic findings: coarse appearing trabecular bone + widening/fraying/cupping of metaphysis of tibia/femur/ulna.

A 2-week old infant has a 102 fever, HR 170 & RR 40. The patient is fussy and the anterior fontanelle is full, but no nuchal rigidity is present. Next step?

-Perform an LP, blood/urine/CSF cultures and admit to the hospital. - Rule out sepsis work-up is initiated. - Realize that nuchal rigidity is not a reliable finding of meningitis until about 12-18 months of age.

A 1-week old infant presents with right midquadrant abdominal mass and decreased urinary output. There has been no temperature lability, irritability, or abnormal stooling or urine appearance. Dx?

-This is likely a Hydronephrosis as urinary tract obstruction is often silent. - In the newborn, a palpable abdominal mass commonly is hydronephrotic or multicystic kidney disease. - do U/S

Two weeks after a diarrheal illness, a 5 year old develops acute onset of pallor, irritability and absent urine output. Dx? Tx?

-This patient most likely has Hemolytic-Uremic Syndrome, the most common cause of acute childhood renal failure. It develops in 5-8% of children with diarrhea caused be EHEC 0157:H7. Treatment is supportive, but some kids may require dialysis. -All kids are followed for potential sequelae of HTN and chronic renal failure.

An adolescent presents with open comedones (blackheads) and closed comedones (whiteheads with purulent debris) on her face and shoulders. She is diagnosed with non-inflammatory acne. First line management includes what?

-Topical benzyol peroxide or a comedolytic agent such as topical tretinoin. -Benzoyl peroxide is bactericidal and keratolytic, causing follicular desquamation (Rx > OTC in efficacy). -Tretinoin is a vitamin A derivative that inhibits comedone formation and increases cell turnover. -Topical, rather than systemic, antibiotics are preferred because of their fewer side effects. Extra: Oral contraceptives are approved for treatment of acne in females. Diet has not been found to have an effect on acne (chocolates, greasy foods are a myth).

An infant with a large PDA typically has a?

....systolic or continuous "machinerylike" heart murmur, an active precordium, and a widened pulse pressure.

Atrioventricular septal defect (also known as AV canal or endocardial cushion defect) consists of a? tx? what is seen?

...contiguous atrial and ventricular septal defect as well as abnormal AV (ie, mitral and tricuspid) valves. b. require correction in infancy to prevent cardiac failure and associated complications. c. systolic murmur of large pulmonary flow is present, and a lower left sternal border diastolic murmur is heard. The second heart sound may be widely split. The chest radiograph and ECG show cardiac enlargement; pulmonary vascularity is increased on the chest film.

Growth hormone (GH) deficiency:

...demonstrate a growth rate that is slow, usually falling away from the normal growth curve (in contrast to constitutional delay where growth parallels the third to fifth percentile curve) b. Bone ages are delayed, indicating catch-up growth potential.

Although individuals with intestinal malrotation may present from birth to adulthood, the classic presentation is that of?

...of an infant with bilious vomiting due to intestinal obstruction. - With prolonged ischemia the bowel becomes necrotic and the patient may have melena or hematemesis, and may develop peritonitis, acidosis, and sepsis.

Acyanotic heart lesions are characterized by shunting of blood from the ___ ___ to ___ ___,

...systemic circulation to the pulmonary circulation ("left-to-right shunt").

Infections involving the teeth, ears, and sinuses may spread to the?

...to the parapharyngeal space, and may ultimately impact neurovascular elements in the lateral space, either because of erosion or mass effect.

Infants with TEF usually present in the newborn period with excessive?

..oral secretions and coughing, choking, and cyanosis secondary to aspirated secretions or with initial feeds. - Infants with the "H-type" fistula (approximately 4% of cases) often present later in life with recurrent aspiration pneumonia or feeding difficulty.

A 16 y.o. male, a resident at a juvenile detention center, was healthy until this morning when he developed a headache, a 105.8 fever, and altered mental status. On exam he has a stiff neck and positive Kernig and Brudzinski signs. He is tachycardic but normotensive. Dx? Confirm diagnosis how? Tx? Acute complications? Most common long term complication?

1) Bacterial meningitis. 2) Lumbar puncture is performed to confirm diagnosis unless pt has a skin infection over the puncture site or has signs of increased ICP. - CSF analysis includes Gram stain and culture, WBC/RBC counts, protein and glucose analyses. (Bacterial meningitis has decreased glucose and increased protein.) 3) IV antibiotics. In suspected pneumococcal meningitis (i.e. caused by Strep pneumo), 3rd g ceph + vancomycin is indicated due to resistance patterns. 4) The MC long term sequelae is deafness, but acute meningitis complications include nerve palsies, hemiparesis, global brain injury, cerebral infarcts/herniations, subdural effusions, venous sinus thrombosis, seizures (1/3 of pts), and SIADH.

1) A newborn has conjuctivitis and staccato cough, and is positive for chlamydia--what do you treat with? 2) A newborn has pneumonia and cultures are pending (PCR for viruses are negative), what do you treat with? 3) A newborn has HSV meningitis, what do you treat with?

1) Erythromycin 2) Ampicillin + gentamicin or cefotaxime. 3) Acyclovir

a. What's the diagnosis if a child establishes his growth curve at or below the 5th percentile by 2 years of age? b. What's the diagnosis if a child has normal growth velocity but is at or below the fifth percentile but not by age 2? c.What's the diagnosis for children who grow at a velocity <5 cm per year and has delayed skeletal maturation?

1) Familial short stature--this is a short child born to short parents will have normal timing of puberty. Bone age = chronological age, no extra growth potential. 2) Constitutional delay of growth--remember "bone age" equals "height age." Growth rate = normal, just late bloomers. 3) Growth hormone deficiency. The screening test for GH deficiency includes serum IGF-1 and its binding protein IGF-BP3.

Kids older than 5 years of age with pneumonia are most infected with what? Tx? If the patient is on a ventilator, what 2 etiologies should you consider?

1) Mycoplasma pneumoniae; Tx is macrolides or cephalosporins. 2) Pseudomonas aeruginosa or fungal species (Candida)

1) Non-organic FTT vs. Organic FTT? 2) Name a common cause of FTT caused by the kidneys and its Tx.

1) Non-organic FTT (most common) is poor growth without a medical etiology, i.e. poverty or poor care-giving. Organic FTT is poor growth caused by an underlying medical (organic) condition; realize this can be assoc. with any organ system. 2) Renal Tubular Acidosis Type 2; give oral bicarbonate.

MR testing?

a. is based on the clinical findings and developmental milestones. b. Other MR testing may include urine and serum amino and organic acids, serum levels of various compounds including ammonia, lead, zinc, and copper, and serum titers for congenital infections.

In DKA, what corrects in first 8-10 hrs? what stays for 24 hrs?

a. low plasma pH and elevated serum ketone levels: will correct significantly in the first 8 to 10 hours b. the serum bicarbonate level may remain low for approx ~ 24 hours or more.

CYSTIC FIBROSIS (CF) triad is?

1. chronic obstructive pulmonary disease 2. pancreatic exocrine deficiency 3. abnormally high sweat electrolyte concentrations.

Signs and symptoms of Neuroblastoma?

1. depend on tumor location: - cervical ganglia tumors may cause Horner syndrome -intrathoracic tumors (most commonly seen in infancy) may be a/w wheezing and respiratory distress - paraspinal tumors may cause compressive neuralgias, back pain, and urinary or stool retention. - Abdominal masses are typically nontender, irregular, and cross the midline. - Metastatic disease typically involves the skin, lungs, liver, and bone. - Bluish skin discoloration (most often seen in infancy) represents subcutaneous infiltration. -Pulmonary involvement can promote increased work of breathing, dyspnea, and pneumonia. - Bone marrow infiltration may cause bone pain and pancytopenia; petechiae, bruising, pallor, and fatigue may occur. -If the orbital bones are involved, proptosis and bluish periorbital discoloration, described as "raccoon eyes," may be noted. -Generalized lymphadenopathy also is common. Some patients develop paraneoplastic syndrome related to tumor neuroendocrine mediators, or opsoclonus-myoclonus syndrome (an autoimmune-mediated phenomenon that may be characterized by cerebellar ataxia).

What can cause dysentery, with fever, abdominal cramps, and bloody diarrhea?

1. enteroinvasive Escherichia coli 2. Campylobacter sp 3. Yersinia enterocolitica - can cause "acute abdomen picture

What are the 3 phases of AD?

1. infant (birth to 2 years) - baby who, during winter months, develops dry, red, scaling cheeks without perioral and paranasal involvement. - chin involved - diaper spared 2. childhood (2 to 12 years): - flexural areas - Perspiration stimulates burning and itching, initiating an itch-scratch cycle. 3. adult (>12 years). - The adult phase begins near the onset of puberty. - flexural inflammation, often accompanied by hand dermatitis - inflammation around the eyes, and lichenification of the anogenital area. - Other findings include keratosis pilaris, accentuated palmar creases, small fissures at the base of the earlobe, and Dennie- Morgan creases under the lower eyelid.

Neonatal HSV disease has?

1. localized skin, eye, and mouth involvement (SEM) - presents @ 1-2 wks of live - requires IV tx 2. CNS disease/encephalitis - at 2-3 weeks of life - fever uncommon - 60% have vesicles - lethargic, irritable, or have seizures 3. disseminated disease with multiorgan involvement. - 1-2 week neonate -fever, lethargy, irritability, anorexia, vomiting, respiratory distress, apnea, jaundice, a bulging fontanelle, seizures (focal or generalized), decerebrate posturing, or coma. - Skin vesicles in two-thirds of cases. -Hepatitis, pneumonitis, shock, (DIC) can occur in severe cases; 30% of do not survive.

What are the 3 categories of CP?

1. physiologic (Pyramidal or extrapyramidal) 2. topographic (hemiplegia, diplegia, spastic quadriplegia) 3. functional (motor quotient)

Galactosemia: is deficiency in what? Symptoms? Management?

11. uridyl transferase deficiency 2. features of jaundice, hepatosplenomegaly, vomiting, hypoglycemia, seizures, lethargy, irritability, poor feeding and failure to thrive, aminoaciduria, liver failure, mental retardation, and an increased risk of E coli sepsis. 3. Management: lactose-free formula.

Describe the expected language development of a normal 12, 18, 24, and 36-month old child (how many words, do they use sentences).

12 months: 2-3 meaningful words + mama and dada. 18 months: A 10-word vocabulary but no combination of words. 24 months: A 50-word vocabulary and 2-word combinations to make a meaningful sentence. 36 months: 250 words producing at least 3-word sentences and use pronouns.

Oligoarticular disease =

= fever than 5 joints involved, divided into persistent and extended categories (disease progressing to affect more than 4 joints after first 6 months) a. MC in toddlers b. serum ANA usu positive. c. Knee mc then ankle. d. 1/4 will get iridocyclitis - opthal screening/slit lamp routinely.

RESPIRATORY DISTRESS SYNDROME:

A condition seen in premature infants resulting from surfactant deficiency. Radiographic findings include a characteristic reticulonodular "ground glass" pattern with air bronchograms and decreased aeration. - Supportive care includes supplemental oxygen as needed to maintain oxygen saturation of 90% to 95% and - intravenous fluids or nasogastric feeding to maintain hydration as the degree of tachypnea usually precludes oral feeding.

symptoms of EBV:

A prodromal period may last for 1 to 2 weeks with vague findings of: fever, nausea, malaise, headache, sore throat, and abdominal pain. - The sore throat and fever gradually worsen - Physical findings during an acute infection may include generalized lymphadenopathy, splenomegaly, and tonsillar enlargement with exudate. -Less common findings include a rash and hepatomegaly. -In small children, many infections are asymptomatic. In others, fever may be the only presenting sign. -Additional acute findings in small children: otitis media, abdominal pain, and diarrhea, Hepatomegaly and rash are seen more often in small children than in older individuals lab findings: -lymphocytic leukocytosis -with ~20% to 40% atypical lymphocytes. - Mild thrombocytopenia is common, only rarely precipitating bleeding or purpura. - . 1/2 pts develop mildly elevated liver function tests, but jaundice is uncommon.

EXTRACORPOREAL MEMBRANE OXYGENATION (ECMO):

A system using a modified heartlung machine utilized in severe pulmonary failure. - Cannulation of the carotid artery and jugular vein is required to link the neonate to the system.

A 3700-g male infant is born full term after an uncomplicated pregnancy. the infant is noted after birth to have a dribbling urinary stream and a lower abdominal mass. Postnatal ultrasound reveals bilateral hydronephrosis with bladder distention. Dx? Most appropriate next test? How do you treat?

A term newborn male with urinary obstruction most likely has Posterior Urethral Valves, and the next most appropriate tests are renal ultrasound and voiding cystourethrogram (VCUG) to confirm diagnosis. Realize PUVs are the most common cause of severe urinary tract obstruction in boys--30% will have renal disease or renal insufficiency even with Tx! Boys who present with UTI are always evaluated for PUV because there's a high association rate (again, obtain renal USG or VCUG). Tx: If it's just PUV with no UTI, relieve the bladder with catheterization and perform endoscopic transurethral valve ablation or vesicotomy.

An 8-year old easily distractible and constantly fidgets. He is a hyperactive boy who cannot complete schoolwork or chores at home. Although the child is very talkative, he does not answer questions clearly. Dx? Next step?

ADHD is suspected--he has inattentiveness, hyperactivity, and impulsivity. Get information regarding his behavior from both the caregiver and from the classroom teacher. He should also undergo developmental and psych evaluations because coexisting psychiatric conditions or learning disability are common--i.e. oppositional-defiant disorder (35%), conduct disorder (25%), anxiety (25%), and depression (20%). Behavioral therapy, classroom modification, and possibly medication can be initiated to treat the child.

EPSTEIN-BARR VIRUS (EBV):

a. A double-stranded DNA herpes virus that infects human oropharyngeal and salivary tissues and B lymphocytes. b. It can cause persistent viral shedding, a.w oral hairy leukoplakia in HIV+ adults and lymphoid interstitial pneumonitis in HIV+ children, and causes several malignancies.

An adolescent male has severe, resistant nodulocystic acne. Treatment? Are there any special considerations for females?

Accutane (isotretinoin) is the treatment of choice. A 4-month course often clears a severe case of acne. It is highly teratogenic and has many side effects including cheilitis (lip inflammation), conjuctivitis, photosensitivity, hyperlipidemia, blood dyscrasias, and elevated liver enzymes. -Rare side effects include depressions, suicide, myalgia, and arthralgia. - Females are required to have a negative pregnancy test before beginning therapy. Intralesional steroid therapy is sometimes used in unresponsive cases.

A 14-year-old Hispanic male presents with a 3-day complaint of "brown urine." - he is a healthy adolescent male with a preceding pharyngitis has periorbital edema and mild hypertension, and has developed tea-colored urine that on microscopy reveals red blood cells. Dx? Lab studies? Prognosis?

Acute Post-streptococcal glomerulonephritis. a. The most important labs to get are serum C3 (low 90% of the time) and C4 (usually normal); b. ASO and Anti-DNase antibodies levels can be checked. APSGN can result from pharyngitis (1-2 weeks after) or from impetigo (3-6 weeks after). c. Prognosis is excellent as ~98% recover completely. EXTRA: Antibiotic use during the initial GAS pharyngitis reduces rheumatic fever risk but does not prevent APSGN.

OM complications:

a. mastoiditis b. temporal bone osteomyelitis c. facial nerve paralysis d. epidural and subdural abscess formation e. meningitis f. lateral sinus thrombosis g. otitic hydrocephalus (evidence of increased intracranial pressure with OM).

A 6-year old boy complains of persistent runny nose. HIs family has tried OTC cold remedies but with marginal, temporary success. His complaints including sneezing paroxysms, itchy throat and eye tearing. He has dark circles under his eyes and a crease across the nose bridge. The nasal turbinates are pale blue, boggy and coated with clear drainage. Diagnosis? Next step in management?

Allergic rhinitis occurs in ~1/3 of kids. To reassure allergic rhinitis Dx, eosinophils in a Hansel stain of nasal drainage can help. Allergy testing may be helpful in patients with severe symptoms. Dx is best managed with allergen avoidance. Nasal steroids ar the most effective chronic allergic rhinitis meds; epistaxis and nasal irritation are side effects. Anti-histamines can control symptoms of sneezing, itching, and nasal drainage. Loratadine is preferred over diphenhydramine because it is non-sedating. Decongestants (pseudoephedrine) are indicated for significant nasal obstruction; side effects include insomnia and agitation. Realize viral and bacterial URIs usually cause congestion with mucopurulent drainage, versus clear rhinorrhea in allergic rhinitis.

A 4-year-old boy presents for a well-child visit. His mother notes that he breathes fast and his lips *turn "dusky" when he runs or plays hard*. The symptoms resolve once he stops the activity. On examination, he has a *II/VI left upper sternal border systolic murmur that radiates to the back; a faint click is heard*. Which of the following is the most likely cause of this child's exercise intolerance?

Although pulmonary stenosis and tricuspid atresia are cyanotic heart lesions, a. exercise-induced cyanosis and b. systolic murmur are characteristic of pulmonary stenosis.

A 16-year old sexually active female presents with 1-day history of stabbing left groin pain, and white vaginal discharge and mild dysuria for 1 week. Last menses was 3 weeks ago with no abnormal bleeding. She takes OCPs and her partners irregularly use condoms. She has suprapubic pain on deep palpation and is afebrile. Next step?

Always perform a UA and a urine pregnancy step as the first step in management. This patient probably has a chlamydial infection causing PID, but others in the DDx include ectopic pregnancy, ovarian cyst, or UTI.

Breast-milk offers antimicrobial properties such as _____ and ______. However, disadvantages of breast-milk include potential HIV transmission, jaundice exacerbation, and low vitamin __ and in ______ & _________.

Antimicrobial properties include IgA (reduces bacterial adherence) and macrophages (inhibit e.coli growth) Breast milk is also low in vitamin D, fluoride and iron, which are usually supplemented at least for the first 6 months of life. Infants should be exclusively breastfed for the first 6 months of life, and then continue to breastfeed for another 6 months but this can be complemented by introduction to solid foods.

A 6-year-old boy presents with his mother after developing acute right groin and knee pain that causes him to limp. No hx of trauma. ROS is positive only for rhinorrhea and sore throat 2 weeks ago. On examination, he is afebrile and his knee exam is normal, but he walks with a right toe-touch gait. Which of the following are the most appropriate next steps in his evaluation?

Assess range of motion of the right hip; measure WBC and ESR; perform ultrasound of the right hip. a. A good rule of thumb is to examine one joint above and one joint below the site of symptoms. b. Hip pain may refer to the groin, anterior thigh, or knee. If the hip's range of motion is near-normal, WBC and ESR are normal, and ultrasound shows a joint effusion, the most likely diagnosis is transient (toxic) synovitis of the hip.

Decrease of GBS why?

B.c of guidelines: recommend screening *women at 35 to 37 weeks' gestation* and offering intrapartum antibiotic prophylaxis to those with risk factors or positive GBS cultures at 35 to 37 weeks' gestation.

A known diabetic goes into DKA. He has had a productive cough and fever the last 2 days. It's important to evaluate this patient for what?

Bacterial infection because it can precipitate DKA.

A 6-month old infant male presents with an abdominal mass discovered by his mom during a bath. On physical, you also notice macroglossia and right-sided hemihypertrophy. Dx?

Beckwith-Wiedemann syndrome with Wilms tumor. This patient is at high risk for developing hepatoblastoma and gonadoblastoma as well!

A full term baby is lethargic and has a HR of 40. O2 was given via bag & mask, and he was intubated but his HR remained at 40. Next step?

Begin chest compressions whenever the HR is still less than 60 bpm despite PPV with 100% oxygen.

The parents of a healthy 12-year-old girl bring her to you for a physical examination required for summer camp. They have no complaints, and the girl denies any problems. Her last menses was normal 2 weeks prior. The camp requires a urine screen. To your surprise, the clean-catch urine screen has significant hematuria. Red cell casts are noted. Parents say dad's side has blood in their urine and they are all doing well. Fam hx = negative for deafness and for renal failure. Microscopy of renal tissue from this patient or from her father will most likely reveal which of the following?

Benign familial hematuria: - an AD condition that causes either persistent or intermittent hematuria without progression to chronic renal failure. - Biopsy = thin basement membrane; in some cases the biopsy is normal.

KERNICTERUS:

a. A neurologic syndrome resulting from unconjugated bilirubin deposition in brain cells, especially the basal ganglia, globus pallidus, putamen, and caudate nuclei. b. Less mature or sick infants = greater susceptibility. c. *Lethargy, poor feeding, and loss of Moro reflex* are common initial signs.

Arsenic symptoms?

a. Acute ingestion of arsenic causes severe GI symptoms b. chronic exposure = skin lesions and can cause peripheral neuropathy and encephalopathy

A healthy, 12-day-old, formula-fed male has jaundice. dx? pathophys?

a. Gilbert syndrome- is a nonphysiologic cause of jaundice, it is a genetic disorder of bilirubin metabolism, involving a decrease in the activity level of uridine diphosphate (UDP) glucuronosyltransferase 1A1.

What is IF?

a. Glycoprotein secreted in the stomach that binds to vitamin B12; b. the intrinsic factor-vitamin B12 complex then attaches to receptors in the distal ileum and is absorbed

CYANOSIS: Central vs. Peripheral

a. Peripheral cyanosis is common in neonates and involves the extremities; it may be normal. b. Central cyanosis is always abnormal and is seen on the tongue, gingiva, and buccal mucosa.

What is the hallmark of intussusception? tx?

a. Peristalsis is still active and attempts to propel contents past the obstruction; this creates episodes of *severe colicky intermittent pain that on subsidence leave the patient calm or lethargic*. b. air contrast enema can be diagnostic + therapeutic

What are 2 life-threatening issues that need to be immediately assessed in Trisomy 21? List some other things associated with DS.

Cardiac defects (endocardial cushion defect 60%, VSD > ToF) and duodenal atresia (upper GI study will show a "double-bubble" sign). Further, hypothyroidism (birth eval), hearing loss (evaluate at 3 mos), and eye strabismus (@ 6mos eval) should also be screened in the first year of life. LONGER TERM DS concerns = ALL, Alzheimer's, and Atlantoaxial instability, obesity, premature aging are all concerns.

A 3-year-old boy with suspected systemic-onset JIA develops tachycardia and dyspnea on the fifth hospital day. He complains that his chest hurts. Heart auscultation reveals a "friction rub" sound. Which of the following is the next best step in management?

Check his oxygenation status through pulse oximetry, obtain a stat electrocardiogram, and consult with a pediatric cardiologist. a. friction rub = pericarditis, common and serious complication of systemic -onset JIA. - best heard along left sternal border - pain relieved when leaning forward - worsened by deep inspiration or coughing - low voltage QRS and ST elevation may be seen - tx = salicylates or steroids.

PARANEOPLASTIC SYNDROME is?

Characterized by hypertension and secretory diarrhea; related to tumor production of catecholamines and vasoactive intestinal peptide.

sepsis x ray shows?

Chest radiologic findings include segmental, lobar, or diffuse reticulogranular patterns, the latter easily confused with respiratory distress syndrome (lack of surfactant).

A 2-year old boy presents with inability to walk for 2 days after falling from the bed onto a carpeted floor. He lives with his mom, 15-month old sister, and 3-month old brother. On PE, the child is apprehensive and has pain on right thigh palpation. X-ray shows a femur fracture. Dx? Next step?

Child abuse. The only trauma history for this child is a fall from a bed onto a carpeted floor; it is unlikely that a significant fall would cause a femur fracture. If a child's development is inconsistent with the injury history, child abuse is suspected.

A newborn has greasy/waxy, yellow-brown scales on his ears, nose, and eyebrows that appeared in the first few months of life. Dx? Tx?

Cradle cap (seborrheic dermatitis). The chronic, symmetrical eruption is characterized by overproduction of sebum. Treatment includes softening the scales with mineral oil, avoiding scrubbing, and daily shampooing with a mild shampoo. Lower potency steroids or ketoconazole shampoo may be helpful.

Acute OM is diagnosed with?

a. fever (usually <104°F [40°C]) b. ear pain (often nocturnal, awakening child from sleep) c. and generalized malaise. d. Systemic symptoms may include: anorexia, nausea, vomiting, diarrhea, and headache. e. Examination findings include a red, bulging TM that does not move well with pneumatic otoscopy.

Infectious mono (EBV other viral) may present with?

a. fever, malaise, adenopathy, splenomegaly, and lymphocytosis. b. Atypical lymphocytes resembling leukemic lymphoblasts are characteristic of these viral illnesses.

An adolescent male presents with significant behavioral changes at home, a decline in school or work performance, and involvement with the law. Most likely Dx? Next step?

DDx = substance abuse, bipolar disorder, brain tumor. Next step = Obtain more history, UDS (urine drug screen), and other commonly assoc drug abuse consequences (like STDs and hepatitis). If negative, pursue psych eval and possible brain imaging.

An infant presents with developmental delay, growth retardation, cataracts, seizures, HSM, and purpura. DDx?

DDx = CMV or Toxoplasmosis. On Head MRI, calcified densities are found periventricularly (CMV) or scattered (Toxoplasmosis).

A 10-year old male presents with weight loss, polydipsia, polyuria, dehydration and Kussmaul breathing. Mom notes he has had nocturnal enuresis for the last week and lately takes big deep breaths to breathe. Dx? Next step?

DKA in a undiagnosed diabetic. Next step is IV normal saline for rehydration, administer insulin, and monitor blood glucose and acidemia. Although the patient's intracellular K+ is depleted, serum levels are high/normal. Note that improvement in acidosis levels is associated with a fall in serum K+. Thus, potassium is added to IVF but only after urine output occurs (No K+ before Pee).

A 2-week old male newborn presents with a "twisted neck." He was born via a difficult vaginal delivery because of his large size (4550 g). On exam, is head is tilted toward the right and his chin rotated toward the left. He has a palpable, firm mass in a contracted right SCM. Diagnosis? Should any imaging be performed?

DX: Torticollis, identified in a patient with an obviously twisted neck with the head and chin point to opposite sides, is commonly due to injury/ contracture of the sternocleidomastoid muscle (SCM) from birth trauma. Torticollis presents soon or after birth; infants may have experienced birth trauma and usually have a palpable, firm mass within the affected muscle. IMAGING: Congenital cervical vertebral malformations can cause torticollis, therefore a cervical spine x-ray should be done. TX: Initially the best treatment is gentle stretching of the SCM if it's a muscular torticollis (as in this case), but if a vertebral anomaly is causing the torticollis stretching the SCM will not help and actually might hurt the patient.

A healthy 2-week old girl has yellow discharge from her left eye. Her mom had early prenatal care and the baby was delivered vaginally. Within the first few days of life, mom noted that the baby had increased tear production in her left eye, which now has yellow discharge. She has red reflexes bilaterally and her pupils are equal and reactive to light (PERRLA). Diagnosis? Next step?

Dacryostenosis (blocked or narrowed tear duct) is considered when infants have unilateral/bilateral increased tearing. It is caused by failure of canalization of the nasolacrimal duct. In almost all cases, dacryostenosis resolves on its own by age 1. Initial treatment involves nasolacrimal massage 3x/day and warm water cleansing. Topical antibiotics are added for purulent discharge. However, note that if the tear duct becomes infected--IV antibiotics is needed (not topical). If the condition persists beyond 1 year, refer to have it canalized surgically.

New parents bring in their daughter in for her 2-week well-child check. Upon asking about pregnancy/delivery history, mom says that it was a C-section because the child was in a breech position. It was mom's first C-section as it was her first child. The baby is otherwise healthy and parents have no immediate concerns. On physical exam, the baby girl has a positive Barlow and Ortolani provocative tests. Dx? What are these tests? and what do they test for? How do you manage this patient?

Developmental Dysplasia of the Hip (DDH)= abnormal relationship between the hip ball & socket joint and femur head. DDH usually results from delivery, but can develop during childhood. If the dislocation persists, the acetabulum will not develop into a cup-like shape and the femur head is forced further out of the socket. Risk factors for DDH: newborn in breech position, positive family history of DDH, females, first-born children. Barlow Test = infant's hips and knees are flexed at 90deg, gentle pressure applied posteriorly; a positive test results when you hear/feel a clunk as the hip disolcates. Ortolani Test = done right after Barlow test, abduct the hip to relocate it into the socket. Ultrasound is most accurate to detect DDH at 6 weeks of age (not useful after 4 months). If a clunk is heard on exam, refer to orthopedics. Most dislocatable hips stabilize w/o intervention within the first 2 weeks of life. If the child is older than 6 months, a Pavlik harness is the best treatment.

cholinergic Excess = DUMBBELSS

Diarrhea Urination Miosis/muscle weakness Bronchorrhea Bradycardia Emesis Lacrimation Salivation/sweating

An 8-year old boy has severe pain with ear movement. He has no fever/n/v/d or other symptoms. He's been in good health and just returned from summer camp where he rode horses and swam. PE reveals a red, tender pinna but a normal TM. Dx? Tx?

Dx is otitis externa (swimmer's ear) and treatment includes administration of topical mixture of polymyxin and corticosteroids. Usually the causative agents are pseudomonas, staph, and occasionally fungi.

TB Treatment

a. initial phase of approximately 2 months' duration on three or four medications, followed by a continuation phase of 4 to 10 months on isoniazid and rifampin. b. Therapy for 9 to 12 months is recommended for CNS or disseminated TB.

PREMATURE ADRENARCHE:

Early activation of adrenal androgens (typically in girls ages 6 to 8 years).... with gradually increasing pubic/axillary hair development and body odor.

PREMATURE THELARCHE:

Early breast development (typically in girls ages 1 to 4 years), without pubic/axillary hair development or linear growth acceleration.

Drooling is a sign of infections of the retropharygneal, parapharygneal, and peritonsillar spaces and what other classic infection? Of the aforementioned abscesses, which one is most common?

Epiglottitis usually by H.influenza (classic thumb sign on lateral x-ray). - Peritonsillar abscess is seen in any age but prevalent in adolescents/young adults. - Peritonsillar abscesses are the most common abscesses seen in pediatrics. Sometimes peritonsillar (and parapharyngeal) abscesses are so bad that patients don't want to even open their mouths (= trismus). - Parapharyngeal infection can impact neurovascular elements in the lateral space. And remember infection in any one compartment can spread to other compartments.

A baby girl came in for her 6-month well check and you notice large coffee-colored macules on the right side of her body. A few days later after receiving a vaccination at her well check, she came in with redness and pain over the injection site and a femur fracture was suspected. At 8 months of age, her mom brought her in right away when she noticed her daughter had vaginal bleeding. On close PE, you notice breast buds as well. Diagnosis?

McCune Albright Syndrome is likely with the given history. MAS is characterized by unilateral cafe-au-lait spots, precocious puberty (i.e. pt's onset of menses), and polyostotic fibrous dysplasia (where your prone for bone fractures).

A male 17-year old male presents with pain and localized swelling over the distal femur causing him to walk with a limp. He plays no sports and has had no recent trauma and the joint is spared. He has no systemic symptoms, other than hemoptysis for 2 days--the reason he is coming in today. You send him for a bone x-ray, with reveals a tumor with "periosteal inflammation" described as a radial sunburst appearance. A CXR was done and a tumor is located in his left lower lobe. Dx? Tx?

Even though rare, osteosarcomas are the most common primary malignancy of bone in children and adolescents. Osteosarcoma is a malignant tumor of bone-producing mesenchyme stem cells, is mainly seen in adolescence with a M:F ratio of 2:1, and has a peak incidence during maximum growth periods. At diagnosis, 20% will have metastases (mostly lung)--but the majority do not and won't have any systemic symptoms. Bilateral Rb is a risk factor. Distal femur + proximal tibia account for the majority of cases (i.e. long bones are affected most). With treatment (amputation or limb-sparing surgery + chemo), at least 2/3 of pts with non-metastatic osteosarcomas will survive, up to 50% of those with lung metastases may be cured.

What is Brachydactyly?

Excessive shortening of hand and foot tubular bones --> box like appearance.

Difference b/w a seizure and febrile seizure?

FEBRILE SEIZURE: A seizure occurring in the absence of (CNS) infection with an elevated temperature in a child b/w age 6 months and 6 years. SEIZURE:Abnormal electrical activity of the brain resulting in altered mental status and/or involuntary neuromuscular activity.

What disorder has defective phosphate reabsorption and deficient conversion of 25(OH)D to 1,25(OH)D in the proximal kidney tubules? Inheritance pattern?

Familial, Primary hypophosphatemia is the most common form of non-nutritional rickets and is X-linked dominant. Children at the age of walking present with leg-bowing, a waddling gait, and short stature. Other findings of calcium deficient rickets are not seen (i.e. rachitic rosary, Chvostek and Trosseau signs).

A 3-year old asian male comes in with 5-days of high fever refractory to tylenol. He has hand and feet edema, a bright right oropharynx, bilateral conjuctivitis, unilateral adenopathy, and what other sign? What sign will he develop in a few days? What sign will he develop in 10 days? What is he at risk for in adulthood? Dx? Tx?

Fine truncal rash; hand and feet desquamation after the edema; thrombocytosis; coronary artery disease. This patient has classic Kawasaki's disease, most prevalent in males younger than age 5. It is a generalized vasculitic disease of unknown etiology but is thought to be infectious. Treatment includes pediatric cardiology consult, early anti-inflammatory therapy with IVIG and high-dose aspirin. In a few weeks, the aspirin dose is changed from an anti-inflammatory dosage to an anti-thrombotic dose and continued for about 2 months when the ESR normalizes. Echo is usually obtained for baseline cardiac function. Even with treatment, approximately 5% of children develop coronary artery dilation, and 1% develop giant aneurysms. Steroids are contra-indicated in treatment.

LEFT-TO-RIGHT SHUNT:

Flow of blood from the systemic circulation into the pulmonary circulation across an anomalous connection, such as a PDA. Such lesions result in pulmonary congestion, but they typically do not cause cyanosis. Systemic hypoperfusion may result if the cause is an obstructive lesion (such as pulmonic or aortic valve stenosis, coarctation of the aorta).

A 6-month old child is fed exclusively goat's milk. What supplements should his PCP advise for him to take? Why?

Goat's milk is 8low in folate/vitamin B12 *and iron which can cause megaloblastic anemia and iron deficiency in the newborn, respectively. Goats are also susceptible to brucellosis thus its milk must be boiled before ingestion. NOTE: Vegan mothers must supplement breast-feeding with vitamin B12.

A 5-year old boy has developed a painless limp over the last 3 months, but no history of trauma. Diagnosis? Explain what this disease is. What's the most serious acute complication?

Most likely diagnosis is Legg-Calve-Perthes disease which occurs often in males 4-8 years of age. A painless limp (or mildly painful) develops insidiously is the most common presenting complaint. LCP disease is defined as avascular necrosis of the femoral head with an unknown etiology. The most serious acute complication is subluxation of the femoral head out of the hip socket.

A 3-year old boy has pallor, lethargy, and decreased urine output. He was well until a week ago, when he had fever and bloody diarrhea that's now resolved. On exam, he is lethargic, has HSM and scattered petechiae. UA reveals hematuria and proteinuria. Dx? What will peripheral blood smear show?

Hemolytic-Uremic Syndrome, characterized by a bout of bloody diarrhea that is followed by a hemolytic anemia, thrombocytopenia, and nephropathy. It is clssically caused by E.coli 0157:H7 > shigella and salmonella. Peripheral blood smear will reveal helmet cells and burr cells.

What is HUS?

Hemolytic-uremic syndrome, the MC cause of acute childhood renal failure, develops in 5% to 8% of children with diarrhea caused by enterohemorrhagic E coli (O157:H7); - usu seen in children < 4 y.o - microthrombi, microvascular endothelial cell injury causing microangiopathic hemolytic anemia, and consumptive thrombocytopenia. -Renal glomerular deposition of an unidentified material leads to capillary wall thickening and subsequent lumen narrowing. - typical presentation = 1-2 weeks after diarrheal illness + acute onset of pallor, irritability, decreased or absent urine output, and even stroke. Kids may develop = edema+ petechiae.

What is the common form of child maltreatment that consists of failure to provide adequate nutrition, shelter, supervision, or medical care?

Neglect. Healthcare providers legally are required to report suspected abuse to CPS (Child Protective Services) or law enforcement, as abuse encompasses physical, emotional, sexual abuse and neglect.

A 7-year old boy has a rash on his lower extremities and pain in his right knee. He has a low-grade fever and abdominal pain. He is non-toxic appearing but has palpable purpura on his legs and buttocks. His right knee is edematous but can bear weight. Blood studies are normal. Dx? Next step?

Henoch-Schlonlein Purpura = a small vessel vasculitis in young children, which classic palpable purpura on the lower extremities and buttocks, abdominal pain, arthralgia, and IgA nephropathy. Know that intussusception is a sequelae to HSP. Next step is to obtain a UA to see the extent of renal involvement. Further, abdominal pain requires careful evaluation.

A previously health 3-year old boy presents with sudden onset of rash. His mother said he was playing outside when she noticed small red spots and a large purple area on his skin. He has no fever, n/v/d, and isn't taking any medications. Three weeks ago he had a viral illness but that resolved. He has neither LAN or HSM. His CBC shows whites at 8500, Hb at 14, and platelets at 20,000. Peripheral blood smear was obtained and is normal. Dx? Next step? Most serious complication?

ITP (Idiopathic Thrombocytopenic Purpura) is suspected when a healthy appearing child develops thrombocytopenia and petechia/purpura. ITP is a condition of increased platelet destruction by circulating antiplatelet antibodies, most frequently antiglycoprotein 2b/3a. Acute ITP is the most common cause of thrombocytopenia in a well child usually aged 2 to 5 years. Young kids usually present with acute onset of petechiae and purpura, and a history of a viral illness a few weeks prior. Within one month of presentation, more than half of all untreated kids have complete resolution. Tx remains controversial. Most serious ITP complication is intracranial hemorrhage, occurring in < 1% of patients. From 10-20% of ITP patients have chronic ITP (aka, lasting >6 months) and are usually older females.

DKA tx pitfalls:

IV fluids with insulin and improvement in acidosis levels causes = FALL in serum K+ levels; addition of potassium to the IV fluids usually is indicated to prevent serious hypokalemia.

What tests are used to determine if a patient is HIV+ in patients younger and older than 18 months?

If <18 months old, PCR is used, detects HIV DNA in wbc's. If >18 months old, use ELISA first then Western Blot to confirm.

This disease is characterized by recurrent painless gross hematuria that is frequently associated with an URTI. When the dark-colored urine is present, there is no dysuria and the pt is otherwise healthy. Diagnosis?

IgA Nephropathy. These patients may develop chronic renal disease over decades. If proteinuria, hypertension, or impaired renal function were present--a biopsy would be necessary.

Two weeks after a viral syndrome, a 2-year-old develops bruising and generalized petechiae that is more prominent over the legs. He has neither hepatosplenomegaly nor lymph node enlargement. Laboratory testing reveals a normal hemoglobin, hematocrit, and white blood cell count and differential. The platelet count is 15,000/mm3. Which of the following is the most likely diagnosis?

Immune (or idiopathic) thrombocytopenic purpura (ITP) is common in children. a. plateletes < 20k b. other labs are normal c. tx IVIG, intravenous anti - D in rh + pts, immunosuppressives or steroids. d. review history for other sources of thrombocytopenia (MMR vax, drug ingestion, HIV).

A child presents with LAN, HSM, weight loss, oral candidiasis, and recurring infections. Dx?

Immunodeficiency. This can be a primary immunodeficiency due to an inherited defect (present at birth) or secondary due to an HIV, malignancy, etc (present after an illness/infection). Always consider immunosuppression in kids that are FTT or have difficult-to-eradicate infections.

When evaluating a patient with possible STI, always ask about dysuria, urinary frequency, presence of discharge, changes in urine appearance, genital lesions, and rashes. If there is a transient pustular rash 4 days after sex and mucoid discharge + dysuria, what's the diagnosis?

In the presence of an STI, a transient pustular rash may be associated with disseminated gonococcal infection. However it is important to know that more than 60% of urethritis patients are asymptomatic, and only 1/3 will have frank dysuria or discharge. Thus, some physicians recommend annual urine screening.

PERINATAL HYPOXIA:

Inadequate oxygenation of a neonate that, if severe, can lead to brainstem depression and secondary apnea unresponsive to stimulation.

A previously healthy 18-month old child has vomiting and severe, colicky abdominal pain. On exam, you find a sausage-like mass. He has not stooled, but you find blood upon digital rectal examination. Dx? Next step?

Intussusception is likely. This vignette is says there's blood on rectal exam; classically, the patient will have currant jelly stools later in the disease course. Order an air contrast enema as it is diagnostic and may be therapeutic as well. Whenever you suspect intussusception, consult a surgeon.

Hereditary angioedema = deficiency of C1 esterase inhibitor; characterized by recurrent, non-pitting edema that runs in families. Bruton's (pan-) Agammaglobulinemia = at 6 months of age, pt will start recurrent infections (esp pneumococcus... pna's and abscesses); all Ig's are low... Tx= replace IgGs Gold standard for Hirschsprungs = biopsy

Iron deficiency is the most common anemia of childhood. Sickle cell anemia will start manifesting after a few months of life (b/c HbF production starts to go down). Diagnose via Hb electrophoresis. Craniopharyngioma: enlarged sella turcica on CT, common cause of short stature. JONES Criteria for Rheumatic fever (i.e. the sequela of Strep throat) = Joint arthralgia, Carditis, Nodules (subq), erythema marginatum (rash), syndenhaam's chorea... if you meet 2 of these, you have RF.

An 11y.o. female has fever, dilated pupils, tachycardia, facial flushing, and hallucinations. Most likely drug?

LSD. LSD is known for it's "bad trips"--patients become terrified and panicked.

ASD usually asymptomatic except when large which may cause?

Large defects may cause mild growth failure and exercise intolerance not appreciated except in retrospect after defect closure

A term newborn infant is admitted to the Neonatal ICU after having a seizure in the Well Baby Nursery. Your examination reveals a microcephalic infant with LBW who doesn't sound. Parent's first child, they recall that they have had fine tremors in their upper extremities and blurry vision. They both also cant smell or taste right. Mom had trouble walking straight in the last few weeks, but she attributes that to her pregnancy. Which of the following environmental toxins is most likely to have caused these findings?

Methyl mercury. 1. Infants exposed in utero to methyl mercury may display: - low birth weight - microcephaly - seizures. - significant developmental delay - vision and hearing impairments. 2. Symptoms in adults + children = ataxia, tremor, dysarthria, memory loss, altered sensorium (including vision, hearing, smell, and taste), dementia, and ultimately death

A 2150-g infant is delivered at 34 weeks' gestation. The mother had prenatal care in Mexico and says she had no problems. Her highest temperature during labor was 100.8°F (38.2°C). The amniotic fluid had a brown-stained appearance. At birth the infant had a diffuse erythematous pustular rash, pallor, poor feeding, tachypnea, and cyanosis. His CBC indicates marked monocytosis. He dies at 4 hours of age, soon after initiation of antibiotics. He most likely had which of the following?

Listeria: - gram-positive rod isolated from soil, streams, sewage, certain foods, silage, dust, and slaughterhouses. - The foodborne transmission of disease is related to soft-ripened cheese, whole and 2% milk, undercooked chicken and hot dogs, raw vegetables, and shellfish. - The newborn infant acquires the organism transplacentally/aspiration/ ingestion at delivery.

Diff b/w SLE and Acute Poststreptococcal Glomerulonephritis?

Lupus nephritis (systemic lupus erythematosus [SLE]) can present similarly and is considered if the hematuria does not resolve or if the C3 level does not normalize in 6 to 12 weeks.

An 8-month old presents with a large red bulge over her femur after she received a shot and cries when you touch it. She had a failed Kasai procedure for biliary atresia earlier in life. What's the MOA for her fracture? Next step?

Malabsorption of vitamin D (and other fat-soluble vitamins) due to lack of intestinal bile salts resulting in Rickets. In addition to an x-ray, next step is to get serum levels: low 25(OH) Vit D, low-normal Ca and Phos, and highly elevated AlkPhos levels.

A full-term 1-week old boy presents with bilious vomiting and lethargy. His mother notes a normal prenatal course and uncomplicated delivery. On physical, he is noted to have significant abdominal distention and blood in his diaper. Dx? Best way to manage?

Malrotation with volvulus. - Malrotation occurs when intestinal rotation is incomplete during fetal development; - volvulus describes twisting of the mesentery of the small intestine leading to decreased vascular perforation which results in ischemia and ultimately bowel necrosis. - Volvulus has a 5-10% mortality rate. -This is a classic presentation for malrotation, an infant with bilious vomiting due to intestinal obstruction. -NOTE: Malrotation there is no colicky pain and the blood is seen per rectum, not in bloody stools. -Best way to manage is emergency surgery to remove any necrotic bowel and to ensure adequate blood supply to surviving intestine--if surgery is not performed, loss of intestine or even death can result. - Initially, order an upper GI contrast series and check fluid/electrolyte status. The upper GI barium will show a "curly Q" pattern.

VIRILIZATION:

Masculinization where infant girls exhibit clitoromegaly, labial fusion, and labial pigmentation; infant boys usually appear normal.

In 30-50% of TEF (tracheoesophageal fistula) patients, other congenital anomalies occur--the most common association being the VATER association. VATER stands for what?

Vertebral abnormality, Anal imperforation, Tracheo-Esophageal fistula, and Radial & Renal anomaly. Be on the lookout if they give you a simple case of TEF, they may ask you what other anomaly might you see (answer will be VATER).

Atrioventricular septal defect left untreated causes?

a, cardiac failure, growth failure, and recurrent pulmonary infections in infancy. b. Pulmonary HTN develops with eventual right-to-left shunting and cyanosis. c. Surgical correction is performed in infancy.

Complications of EBV

Neurologic sequelae include: - Bell palsy, seizures - aseptic meningitis or encephalitis - Guillain-Barré syndrome - optic neuritis, and transverse myelitis. -Parotitis, orchitis, or pancreatitis may develop. - Airway compromise may result from tonsillar hypertrophy; treatment may include steroids. -Splenomegaly is seen ~1/2 those with mono, rupture is rare, but the blood loss is life-threatening.

12-month old girl has poor weight gain, but physical exam is normal. Next step? Dx?

Next step = Obtain patient history, especially dietary and social histories. Thereafter, lab screening tests, dietary counseling, and close weight-watching are appropriate. Dx = Failure To Thrive (FTT): suspected when growth is below the 3rd to 5th%, or if growth drops more than 2 major percentiles in a short time.

An 8-year-old child presents to the emergency department with the complaint of right-sided weakness. Hx of Sickle Cell Disease. Next step?

Next step: ICU + exchange transfusion to reduce the amnt of circulating sickle cells, reduce neurologic damage. Long term strategy: 2nd stroke in upcoming 2 years = 70%-80%, so chronic transfusions needed.

A small-appearing, 5-year-old girl previously diagnosed with asthma, rectal prolapse, and sinusitis presents with fever, scant purulent rhinorrhea, abnormal breath sounds, and digital clubbing. Next step?

Obtain family history and order a sweat test and chest x ray. The defective protein in Cystic Fibrosis is the Delta-F508 (deletion of phenylalanine at amino acid 508) on chromosome 7. CF patients have such thick mucus that it causes a cycle of obstruction, inflammation, and infection. Further, these patients have pancreatic exocrine deficiency and boys have azoospermia. CF patients initially will get a Staph Pneumonia (PNA) then Pseudomonas PNAs. A negative sweat test doesn't exclude CF.

A newborn baby was born at home. On day 2 of life, he has puffy eyelids, copious purulent ocular discharge, chemosis (big blister on eye), and red conjuctivae. Most likely diagnosis?

Ophthalmia neonatorum refers to conjuctivitis occuring within the 1st month of life. a. Days 2-5, most likely is gonorrhea conjuctivitis. b. Days 5-14, most likely is chlamydial conjuctivitis. tx: erythromycin (has risk of causing hypertrophic pyloric stenosis) c. Chemical conjuctivitis is self-limited and presents within 6-12 hours of birth as a result of ocular silver nitrate or erythromycin prophylaxis irritation.

A 2-year old developed emesis and intermittent abdominal pain yesterday. His parents weren't overly concerned because he did okay between pain episodes. Today however he has persistent bilious emesis, several bloody stools, and a vague tubular mass in the RUQ. He is febrile, tachycardic, and lethargic. Next step?

Order a contrast enema. This patient probably has intussusception, evidenced by his colicky abdominal pain, currant jelly stools, and a "sausage-like" mass in the RUQ (near ileocecal junction). They often prefer to have their knees flexed to the chest. Consider malrotation with volvulus as well. Often these patients are infants who will have abdominal distention and blood per rectum (not passing stool) with bilious vomiting.

A previously healthy 12-year old boy has had right knee pain for 3 weeks. He is athletic, playing basketball and track, but he denies any recent trauma. He describes increased pain when he runs or jumps. Physical exam notes mild edema and tenderness over his right tibial tuberosity. Dx? Next step?

Osgood-Schlatter Disease is a clinical diagnosis. Next step for most patients is rest and ice after activity + NSAIDs prn. His lack of constitutional symptoms (fever, joint erythema, weight loss etc) are clues that this a relatively benign condition. The knee pain of OSD is caused by inflammation of the tibial tubercle, an extension of the tibial epiphysis (aka growth plate). Ossification centers begin to form between ages of 9 and 13 and are completed at the completion of puberty. Repetitive running/jumping cause traction and microfractures to the developing area resulting in inflammation, edema and tenderness.

Difference b.w Papule, Nodule and Pustule?

PAPULE: Small, erythematous, and inflamed "bump" under the skin due to sebum, fatty acids, and bacteria reacting within a follicle. NODULE:Papule greater than 5 mm penetrating deep into the dermis. PUSTULE: Elevated focus of inflammation and purulent exudate around a comedone, occurring in the superficial dermis.

A newborn baby is delivered at a rural hospital where little prenatal and newborn screening is performed. The baby comes in for 1 month check-up and has fair skin, fair hair, blue eyes, and a musty odor to him. The doctor notices eczema periorally on the baby, but the diaper area is spared. The doctor sends him to the city hospital to get a screen done, as he is suspecting what?

PKU (phenyl-ketonuria), an autosomal recessive disease.. The most common symptom in these children is mental retardation, among the others listed in the question stem.

Patients having encephalopathy or encephalitis after receiving the diphtheria, tetanus, and pertussis vaccine do not receive ____doses.

Patients having encephalopathy or encephalitis after receiving the diphtheria, tetanus, and pertussis vaccine do not receive *subsequent* doses.

A 7-year old girl undergoes Tanner stage 2 development. The girl complains of sharp pains in her lower "belly." On physical exam suprapubic abdominal mass is felt on examination. Most likely diagnosis?

Precocious pseudopuberty (gonadotropin-independent thus no H-P-G axis activation). With the given history/PE, an estrogen-producing tumor of the ovary or adrenal gland is highly likely--other causes of precocious pseudopuberty includes exogenous sources of estrogen like birth control pills or estrogen creams.

Children with ASDs are often asymptomatic and are discovered on routine exam. Sometimes with large ASDs, growth failure and exercise intolerance may be seen; usually ASDs are well tolerated during childhood but can lead to what problem in adulthood?

Pulmonary Hypertension. Extra Cardiac Pearls: Acyanotic heart lesions are characterized by shunting of blood from left-to-right (systemic to pulmonary). Know these shunts can reverse overtime and cause cyanosis if pulmonary hypertension develops (Eisenmenger syndrome).

OPSOCLONUS-MYOCLONUS SYNDROME:

SYNDROME: Characterized by chaotic eye movements and myoclonic jerks; described as "dancing eyes, dancing feet" related to autoantibodies produced against neuronal elements

Hearing loss caused by cochlea damage due to infections, noise, ototoxic agents, or genetic defects is classified as what time of hearing loss? What's the most common autosomal dominant syndromes associated with this condition? What's the most common infectious cause of this condition?

Sensorineural Hearing Loss. It can be congenital or acquired, with about half of cases resulting from genetic factors. The most common AD syndromes associated with SNHL include Waardenburg syndrome types 1 and 2, and brachio-oto-renal syndrome. Waardenburg patients have a white forelock, heterochromic irises, and deafness. BOR syndrome has branchial fistulas, pre-auricular pits, hearing loss, and renal impairment. Prenatal CMV infection is the most common infectious cause of SNHL; it can cause hearing loss in late infancy/early childhood. However, other TORCHES infections can cause SNHL too. Note: ototoxic drugs include cisplatin, loop diuretics, and aminoglycosides.

A 4-year-old boy has started growing pubic hair and has recently exhibited aggressive "bullying" behavior at his preschool. Physical examination reveals height and weight above the 95th percentile, marked muscular development, Tanner stage II pubic hair development, scant axillary hair, prepubertal testicular size, a masculine voice, and oily skin. The abdominal examination is normal. The child's bone age is 6 years. Which of the following is the most appropriate next step in management?

Serum 17αhydroxyprogesterone level. Boys with congenital adrenal hyperplasia have virilization despite prepubertal testicles. --> This results from a disorder of steroid synthesis, leading to a deficiency of cortisol and an overproduction of androgenic intermediary metabolites such as 17αhydroxyprogesterone.

A 16-year old female presents with persistent vaginal bleeding. She has been seen 3 months ago when she had a mild anemia of 13, diagnosed her with DUB and started her on iron. Today she is listless and pale with a Hb of 6 and negative pregnancy test. She is admitted and transfused with PRBCs (packed RBCs). In addition to stabilizing her circulatory system, what else should be done to stabilize her?

Since her Hb is 6 she is classifed as with severe DUB. In addition to the transfusion, administer IV estrogen (Premarin) and high-dose combination OCP to stop the bleeding.

A 7-month old presents with misalignment of the eyes, as her left eye deviates inward. Dx? How do you make the diagnosis?

Strabismus = misalignment of the eyes. Specifically, this is an esotropia because the eye deviates inward. Diagnosis is made using the corneal light reflex or cover test. The cover test is as follows: the patient covers 1 eye and fixes on an object, the pt removes the cover and a positive test will show eye drift. Screening for stabismus by means of cover testing should be included in every pediatric check-up. Early medical and surgical intervention results in an improved chance for normal alignment and acuity. If strabismus is not picked up, amblyopia can result.

What is seen with rhabdomyolysis?

Strenuous activity can cause rhabdomyolysis and dark urine, but patients with these conditions often will have muscle aches, fatigue, nausea and vomiting, and fever.

Name the three most common pathogens that cause AOM. What does the characteristic AOM tympanic membrane look like? Treat with what?

Strep pneumo > non-typeable H. flu > moraxella. The characteristic TM is red and bulging, and does not move well with pneumatic otoscopy. Treat with amoxicillin 80-90mg/kg/d and pain medication prn for 7-10 days.

What age groups are at risk for subdural hemorrhage? epidural hemorrhage? Compare and contrast subdural and epidural bleeds: bilateral/unilateral? seizures/no seizures? mortality/morbidity? appearance on CT?

Subdural: - children younger than 1 year and the elderly. - Most subdural bleeds are bilateral occuring in the supratentorial space, -have assoc-seizures 75 %, and are associated with high morbidity b/c brain parenchyma is affected. - retinal hemorrhages - Also, 1/3 of all pts have skull fracture. On CT, subdural appears concave like a crescent-moon. - can be chronic > 21 days, acute, subacute.(3-21 days) Epidural: - older kids and adults. - 2/3 a/w skull fx - rapid in progression, require surgical intervention. - A characteristic lucid interval is experienced before rapid decline. - Although most adult epidural hemorrhages are arterial in origin, in children ~half originate from venous injuries. - The middle meningial artery is often ruptured in the infratentorial space. -Most epidural bleeds are unilateral, do not have seizures, and have high mortality. -On CT, the bleed is a biconvex hematoma.

A mom and her son were going for a walk in the park. They parked the car and were about to cross the street when a car flew by. The mom gave a sudden, strong jerk of the child's hand to stop him from crossing the street. The child now dangles his affected arm close to his body, with the elbow slightly flexed and the forearm pronated. Dx? Treatment?

Subluxation of the radial head (Nursemaid's elbow). Treatment consists of reducing the dislocation by extending the elbow and pronating the hand. A successful reduction is accompanied by a click as the entrapped annular ligament pops into place. Usually the child begins to move the arm within minutes.

Three diverse entities fall under the JRA rubric, classified according to symptoms occuring in the first 6 months of illness: *systemic, polyarticular and pauciarticular. * Define each of them (i.e. how many involved joints, are there systemic symptoms or not).

Systemic disease has systemic symptoms and classically presents with a. daily *high-spiking fevers for 2 weeks or more b. a rash and arthralgias that wax and wane with the fever c. lymphadenopathy, and organomegaly*.

In male puberty, what initiates the order of pubertal events: pubic hair growth, penile lengthening, testicular enlargement, or attainment of maximal height velocity? In female puberty, what initiates the order of pubertal events: pubic hair growth, breast budding, menarche, or attainment of maximal height velocity?

Testicular enlargement is the first pubertal even that occurs in males, followed by the other events in no specific order. Breast budding (thelarche) is the first pubertal even that occurs in females, followed by the other events in no specific order.

On cardiac exam, review APT M. - Diastolic "ARMS". - Systolic murmurs radiate (Aortic stenosis radiates to carotids, Mitral regurgitation radiates to axilla). -What are the 5 "blue baby"/early cyanotic defects/right-to-left shunts in a newborn?

The 5 T's + vowels: Transposition of the great vessels, Tetralogy of Fallot, Tricuspid Atresia, Total Anomalous Pulmonary Venous Return, Persistent Truncus Arteriosus

Pediatric tuberculosis = symptoms?

a. *Neck and perihilar or mediastinal lymphadenopathy* b. pulmonary or extrapulmonary manifestations can occur, with miliary disease and meningitis MC in infants and younger children. c. *Fever, weight loss, and lower respiratory tract signs and symptoms* (possible left pleural effusion in this patient) are archetypal TB findings. d. Tx: PPD, negative pa isolation

What prenatal measures could a mother receive to check for Trisomy 21? What are the findings?

Triple screen at 15th and 20th weeks: The AFP and estriol would be 25% decreased and hCG doubled. Amniocentesis and chorionic villous sampling can provide chromosomal analysis. Ultrasound can be performed and show increased nuchal translucency.

Ebstein Anomaly:

Triscuspid valve of Ebstein anomaly = insufficient because leaflets = displaced inferiority into RV --> smaller ventricle chamber and obstructs ventricular outflow.

A newborn has scalp *cutis aplasia* (missing portion of the skin and hair), *polydactyly*, holoprosencephaly, small HEJ (head, eyes, jaw), and a sloping forehead. Dx?

Trisomy 13 (Patau Syndrome). Realize a single umbilical artery and cleft lip/palate are common to both Trisomy 13 and 18.

Clinical Pearls

The etiology of pneumonia varies according to the patient's age. a. Neonates have the greatest risk of group B Streptococcus, b. toddlers are more likely to have respiratory syncytial virus c. adolescents usually contract Mycoplasma.

Symptoms of DKA?

The history often is positive for: - polyuria - nausea, vomiting - abdominal complaints. - Hypothermia - hypotension - Kussmaul respirations, - acetone on the breath are common.

An 8-year old girl has a positive NAAT (nucleic acid amp test) for N. gonorrhea. She discloses that her mother's boyfriend sexually abuses her. Management? Next steps?

The test for N.gonorrhea first must be confirmed by a second NAAT that amplifies a different gene sequence or by culture, and since there was genital-genital contact--other STD testing is warranted (HIV, chlamydia, trichomonas, syphilis). CPS or law enforcement MUST be notified in abuse cases. Gonorrhea treatment = 1 dose of IM ceftriaxone.

A newborn has a single umbilical artery, inguinal hernia, a weak cry, clenched hands with overlapping digits, a small jaw and mouth with a high arched palate, and rocker-bottom feet, cardiac defects: (VSD, ASD, PDA or coarctation of the aorta), small palpebral fissures, prominent occiput. Dx?

Trisomy 18. A weak cry and the clenched hands are good clues that it is Edward's syndrome.

What are Tet spells?

a. *sudden increase in right-to-left shunting of blood* b. These spells may be brought on by activity or agitation, or they may occur without an apparent precipitant. c. Such children --> squatting posture, which compresses peripheral blood vessels, thus increasing pulmonary blood flow and systemic arterial oxygen saturation.

A 14-month old girl has a 6-hour history of fever to 102.6 and an increasingly ill appearnce. She is anxious and doesn't want to leave her mom's arms, but only gives a faint cry when approached. Her RR is 70 and her neck is hyperextended. An area of slobber is noted on mom's blouse. Next step?

This baby girl sounds like she has epiglottitis, a medical emergency, as evidenced by the hyperextended neck, faint cry, and drooling. The next step is to arrange for immediate transfer to the operating room to secure the airway via tracheal intubation or tracheostomy.

A previously healthy, 4-year-old boy has been febrile for a day. He does not want to drink and vomited this morning. There have been no URI symptoms or diarrhea. On examination, he is sleepy, but arousable, and has a temperature of 102.8°F (39.3°C). His posterior oropharynx is markedly erythematous with enlarged, symmetrical, and cryptic tonsils that are laden with exudate. Shoddy cervical lymphadenopathy is noted. He moves his head vigorously in an effort to thwart your examination. Which of the following is the next best step in your evaluation?

This child has a fairly classic examination for streptococcal tonsillitis. - The potential for a retropharyngeal or peritonsillar process is diminished by the lack of tonsillar asymmetry, soft palatal changes, and nuchal rigidity. - A rapid streptococcal immunoassay would be a good initial test; a swab for culture may be sent as well. - Standard therapy would include oral or intramuscular penicillin in the nonallergic patient and an analgesic/antipyretic. - If the streptococcal immunoassay is negative, some treat patients whose H&P are consistent with streptococcal infection while awaiting culture.

A 12-year-old boy requires a sports physical examination. He denies chronic health problems, including adverse exertion symptoms. a. The clinician notes a I to II/VI left upper sternal border systolic murmur that does not radiate. b. The second heart sounds splits normally, and no audible click is appreciated. Peripheral perfusion is normal, and the fingers are not clubbed. dx?

This child has a* benign pulmonary flow murmur*, differentiated from a pathologic pulmonary murmur in that it does not radiate, no click is heard, and no signs and symptoms of cardiac disease (digital clubbing, cyanosis, exercise intolerance) are found.

A 7-month old boy has respiratory difficulty and is brought to the ER at 3am. His mom says he first developed a cough 2 days ago, and the cough has become "barky." On exam he has a low-grade fever, tachypnea, and tachycardia. A moderately inflamed pharynx and inspiratory stridor are noted on PE. Next step?

This child likely has croup (croup is worse at nighttime and especially on the 2nd night). Administer aerosolized racemic epinephrine and corticosteroids immediately to reduce laryngeal edema. Cool mist is often used in an attempt to relieve laryngeal spasm.

An obese 13-year old African-American male complains of right knee pain. He denies any recent trauma, but has a limp favoring his right lower extremity. What should you evaluate for? Order what tests*?

This child probably has Slipped capital femoral epiphysis (SCFE) as he has several risk factors for it: pubertal male, obesity, and African-American. A decrease in range of motion of the hips will be found on PE, and know that hip pain may be referred to the knee. SCFE is the gradual or acute separation of the proximal femoral growth plate with the femur head. X-ray with the child's hips in the frog-leg lateral position are the study of choice for epiphyseal displacement. Order thyroid, LH and FSH levels -- think of gonadal deficiencies in these patients.

An adolescent male presents with dysuria and urethral erythema with scant mucoid discharge. No other findings, no increased urinary frequency. Diagnosis? Possible etiologies?

This is a Urethritis caused by a STI. Most common causes are gonorrhea (2-5 days incubation, very purulent) or chlamydia (5-10 days incubation, less pus). Urine PCR testing for GC and chlamydia have great sensitivity and is less invasive than swab cultures--however, if the pt is a victim of abuse, urethral swabs are required. NOTE: Urine leukocyte esterase+ means bacterial infection present when there's pyuria; Urine Nitrite+ means the infxn is caused by an Enterobacter (i.e. E.coli)--for a positive nitrite test >100,000 colonies need to be grown.

A 19-year old female presents with cervical adenopathy. On PE it is painless, rubber-like, and about 2cm large. On further examine, you notice she also has mediastinal lymphadenopathy. She tells you she wakes up in night sweats once every couple weeks but attributed it to the warm temperature of her dorm room. The doctor is worried about what right now? What imaging/lab tests should he do?

This is a classic presentation for Hodgkin's Lymphoma, where the most common presentation is painless, rubbery cervical adenopathy in 80% of pts while 70% also have mediastinal lymphadenopathy. Lymph nodes that are >1cm and are nontender are of great concern. Also, drenching night sweats and unintentional weight loss are systemic clues to malignancy as well. The cause of HD is unknown, but seems to be multifactorial. HD has an association with EBV as well. The first thing to do is get a chest radiograph. Then, node biopsy is required to make the diagnosis; classically, Reed-Sternberg cells will be seen.

A 36y.o. woman with little prenatal care delivers a AGA baby girl that has hypotonia, upslanting palpebral fissures, epicanthal folds, redundant nuchal skin, and fifth finger clinodactyly. Dx? Next step?

Trisomy 21 (Down Syndrome- Amar). The baby may also have brachydactyly with an associated single palmar crease, Brushfield iris spots, and a flat face. Next step = obtain chromosomal analysis for confirmation, provide counseling, and evaluate for other features of the syndrome.

If you're allergic to egg, what shots can't you receive? What shots can't be given to pregnant or immunocompromised patients?

True contraindications to receiving immunizations are rare, but do include immediate hypersensitivity to vaccine components such as egg. Allergy to egg is a contraindication to influenza and yellow fever but NOT (MMR). In general, live virus vaccines are not given to preggos/immunocompromised patients

True or False: Conditions that are not contraindications for vaccines are mild URI, gastroenteritis, and low-grade fever.

True. These are all not contraindications to receiving a vaccine.

Chronic subdural hematomas MC in? symptoms?

a. ...more common in older children than in infants; b. symptoms may include: chronic emesis, seizures, hypertonicity, irritability, personality changes, inattention, poor weight gain, fever, and anemia. c. Magnetic resonance imaging is more useful than CT for evaluating subacute and chronic hematomas because the hematoma age can be estimated by signal intensity.

Tx for lead poisoning?

Treatment varies depending on the BLL and the patient's symptoms. Admission to the hospital, stabilization, and chelation are appropriate for symptomatic patients

A 15 y.o. female with primary amenorrhea, < 5th% for height, and a low hair hairline. At birth, she had edema of the neck and feet. Dx?

Turner's syndrome. These patients can also have coarctation of the aorta, a horseshoe kidney, and a broad chest. Mental development is normal. A common finding is weak or absent lower extremity pulses due to the aortic coarctation!

symptoms of shigella:

Typically, children have: - fever, cramping abdominal pain, watery diarrhea (often progressing to small bloody stools), and anorexia. - Untreated, diarrhea typically lasts 1 to 2 weeks and then resolves. - Neurologic findings may include HA, confusion, seizure, or hallucinations. - Shigella meningitis is infrequent. - Uncommon complications include rectal prolapse, cholestatic hepatitis, arthritis, conjunctivitis, and cystitis. - Rarely, Shigella causes a rapidly progressive sepsis-like presentation (Ekiri syndrome) that quickly results in death.

A 2-month old girl presents with fever and vomiting. UA comes back for positive nitrates and leukocyte esterase. Urine culture confirms UTi; renal USG and VCUG show mild hydronephrosis and grade II VUR on the right. Next best step?

UTI will be treated with antibiotics. Thereafter, infants and children with VUR will receive prophylactic antimicrobial therapy (sulf-trimeth, trimeth alone, or nitrofurantoin) and close monitoring for infection with UA and urine culture ever 3-months.

Cyanotic CHD is characterized?

Unsaturated blood returning to the heart from the periphery = shunted into the systemic circulation = lungs bypassed

An 8-year old boy presents with bedwetting 3-4x/week for as long as he can remember. He has a strong urinary stream, day time continence, and no UTIs. Mom says he can sleep thru anything. Physical exam is normal. Next step in treatment?

Use an enuresis alarm for primary nocturnal enuresis, which is successful almost 90% of the time. Medications that can be used include night-time doses of imipramine or oral DDAVP (an ADH analog).

UA of acute post strep glomerulonephritis shows?

Urinalysis typically reveals high specific gravity, low pH, hematuria, proteinuria, and red cell casts.

For full-term infants without hemolysis, phototherapy is initiated at the following bilirubin levels:

a. 16-18 mg/dL at an age of 24 to 48 hours b. 16-18 mg/dL at 49 to 72 hours; c. and more than or equal to 20 mg/dL at 72 hours or more

Women are screened for gestational diabetes (GDM) between?

a. 24 and 28 weeks of pregnancy (but can be screened earlier if considered high risk). b. many of the congenital malformations a/w gestational diabetes are thought to result from *hyperglycemia early in the pregnancy*.

Male pseudohermaphroditism:

a. 46,XY karyotype b. causes: - testosterone dyssynthesis - 5α- reductase/dihydrotestosterone deficiency - decreased androgen binding to target tissues (AIS - mc form) -phenotypically normal F with functioning testicular tissue, variable incomplete virilization of genitalia, and short, pouchlike vaginas; - typically diagnosed at puberty when primary amenorrhea noted; - maintain as females and offer vaginoplasty

Rales:

Wet or "crackly" inspiratory breath sounds due to alveolar fluid or debris; usually heard in pneumonia or congestive heart failure (CHF).

Most VSD's close spontaneously by? Medical Management?

a. 6 to 12 months of life. b. Medications= diuretics (eg, furosemide, chlorothiazide) and afterload reduction agents (eg, an angiotensin-converting enzyme inhibitor) and sometimes digoxin. c. infants also need caloric intake, usually 140 kcal/kg/d, and may require feeding tube.

PULSUS PARADOXUS:

a. A blood pressure that varies more widely with respiration than normal. b. A variance of greater than 10 mm Hg between inspiration and expiration suggests obstructive airway disease, pericardial tamponade, or constrictive pericarditis.

A 3-month old infant is found in the morning not breathing. CPR was initiated by parents and continued by the paramedics. He was pronounced dead in the hospital. What do you do next? Diagnosis? Next step?

What you do first is say "besides everyone's best efforts, their child has died." Ask the parents if they would like you to call anyone for them (friend, family, clergy etc). a. Most likely diagnosis is SIDS, Sudden Infant Death Syndrome. After a period of grieving, let the family know protocol for SIDS deaths--A coroner must perform autopsy and police must exam the home for clues. Emphasize that these measures can help to bring closure for the family and may yield important information for preventing future child deaths should they have kids. b. Know that apnea of infancy does not increase the risk of sudden death, and the use of home apnea monitors do not reduce the risk of SIDS.

Clinical Pearls from Dr. Eduardo Pino: Limps --> other than trauma, separate them into groups: DDH (0-3yrs), LCP (4-8yr), SCFE (adolescents). Painless, non-pulsatile mass on the back of the knee = Baker's cyst --> benign, just give parents reassurance. Pt is on antibiotics for a while and gets diarrhea = C.diff, order stool test for it.

Whole milk is only good for baby cows or babies over 1 year. The majority of patients that are obese are due to eating too much, not glandular problems. Most newborn reflexes disappear by 4-6 months of age. Parachute reflex never goes away. Pincer at 9 months age. 3 cubes at 15mo, 4 cubes at 18, 7 at 24 months. Social smile at 4 weeks of age.

A mom feels a mass in the belly of her 4-year old son during a bath. He has no significant history. His BP is 130/88, HR 82, and has both pallor and a firm left-sided abdominal mass that does not cross the midline. Dx? next step?

Wilms tumor. Next step: obtain abdominal imaging, check UA for hematuria, BMP for renal/hepatic dysfunction, and CBC for anemia. Note that patients with Wilms tumor are slightly older and appear less sick than neuroblastoma patients.

An institutionalized male juvenile deliquent has nodulocystic acne, a prominent glabella, an explosive temper, and a low-normal IQ. Dx?

XYY male. By age 6, these patients tend to be taller than their peers and begin having defiant behavior.

A healthy-appearing term infant suddenly loses interest in feeding at 12 hours of life and develops cyanosis, poor peripheral perfusion, and tachypnea. Physical reveals only a loud second heart sound and no murmur. Best initial management? Dx?

a. Administer prostaglandin E1 to maintain patency of the ductus arteriosus...then do atrial septostomy (later) via cath lab. b. Patient likely has a cyanotic congenital heart defect (i.e. transposition of the great vessels). In the first hours of life, the ductus arteriosus and the foramen ovale provide a connection between pulmonary and systemic circulation--once the DA closes, symptoms appear. c.TGA has an "egg on a string" appearance on CXR and is incompatible with life.

Atopy (allergies) and a family history of asthma are strong risk factors for asthma development. Further, 40-50% of kids with RSV bronchiolitis later develop asthma. What's the median age that kids get asthma?

a. Age 4. Asthma can be triggered by dust mites, animal dander, cigarette smoke, weather changes, URIs, beta antagonists, some nsaids and exercise. Two to four hours after acute exposure, a late-phase reaction (LPR) begins. b. The LPR is characterized by airway inflammation and bronchoconstriction. Asthma is different than COPD because asthma is a reversible process.

A 2-month-old boy has 3 days of stools with blood streaks intermixed in them. Over the past week, he has been stooling more often with an increase from 4 stools to 8 stools per day. He has been happy and afebrile, and no blood is found when she wipes him. He continues to take 3 ounces of standard infant formula at each feed.

a. Allergic proctocolitis is induced by allergy to the protein in cow's milk (found in std infant formulas), soy protein is similar in structure so cross allergy exists. b. if protein not removed from diet =infant can progress to enterocolitis with resulting *severe diarrhea, malabsorption, vomiting, and dehydration*.

Evaluation of the infant with ambiguous genitalia must occur rapidly to alleviate family anxiety....what is the intersex eval team?

a. An endocrinologist, clinical geneticist, urologist, and psychiatrist are essential members of the intersex evaluation team. b. The goals = find etiology of the intersex problem, assign gender, and intervene with surgical or other treatment as soon as possible.

Diagnosis of ALL?

a. BM examination b. normal marrow contains less than 5% blasts, minimum 25% blasts confirm diagnosis. c. 2/3rd children with ALL have leukemic cell karyotypic abnormalities: in chromosome number (ie, hypodiploidy or hyperdiploidy) or chromosome structure (translocation, deletions, inversions).

What is the difference in onset of bacterial pneumonia versus viral pneumonia? Compare bacterial/viral/mycobacterial pneumonia CXRs.

a. Bacterial PNA progresses rapidly over a few days with high fever and is often lobar on CXR, also Pleural effusion and abscess formation more consistent with bacterial. b. Viral infection may develop more gradually and CXR shows air trapping with flattened diaphragms. c. Finally, mycobacterial PNA (atypical PNA) will have an interstitial pattern.

Lead poisoning signs:

a. Behavioral signs of lead toxicity include hyperirritability, altered sleep patterns, decreased play activity, loss of developmental milestones (especially speech), and altered state of consciousness. b. Physical symptoms include vomiting, intermittent abdominal pain, constipation, ataxia, coma, and seizures.

Difference b/w Chancre and Chancroid?

a. CHANCRE:Painless ulcer with indurated base usually caused by Treponema pallidum. b. CHANCROID: Painful ulcer with exudate caused by Haemophilus ducreyi, tender inguinal lymphadenopathy also noted. c. HSV lesions = may start with localized pruritus, not typically seen with other STDS.

An 8 y.o. girl has had a headache for 2 weeks and intermittent fever up to 101. She has had frequent AOM and sinusitis episodes, but her last episode was 5 weeks ago. She started vomiting non-bloody, non-bilious fluid a few days ago and appears lethargic. She has frontal sinus tenderness and nuchal rigidity. Next step?

a. CT of the head because this child likely has a intracranial mass, probably an abscess from her recurrent sinusitis infections b. In her case, CNS imaging (with contrast) is performed prior to an LP, if it was a mass lesion causing ICP= and u do LP can cause herniation of the brain.

In a work-up for a child with ALL, what other procedures/imaging should you get? What is the treatment?

a. CXR--see if any mediastinal involvement (more prominent in T-cell). b. Lumbar puncture--examine CNS for any leukemic involvement. c. Bone x-rays--if abnormal findings, worse prognosis. e. Induction therapy for 4 weeks = glucocorticoids, asparginase, vincristine. f. Maintenance tx for 2-3 years = MTX, 6-MP, glucocorticoids and vincristine.

NICOTINIC SYMPTOMS:

a. Cardiac (hypertension, tachycardia, arrhythmia) b. muscle (fasciculations, weakness, tremors) c. respiratory failure due to diaphragm paralysis d. hypertension.

A 5-year old girl is brought to your clinic because she has developed breast and pubic hair over the past 3 months. Physical exam reveals a girl whose height and weight are >95%, Tanner stage 2, oily skin and facial acne. Dx? Next step?

a. Central precocious puberty-- b. Obtain more history, FSH & LH serum levels, and bone age radiographs. c.True (central) precocious puberty is gonadotropin-dependent, meaning the hypothalamic-pituitary-gonadal axis is activated too early. A central nervous system (CNS) cause of true precocious puberty must be ruled out because she is younger than 6 to 8 years, and a CNS cause must be ruled out in boys at any age below ~9 years. d. also she may have precocious (non-central) pseudopuberty

Cerebral palsy is a non-progressive disorder of movement and posture that results from* antenatal insults* to the immature CNS (thus it is not a peripheral neuropathy or myopathy). Approximately how many pts are mentally retarded? how many have seizures?

a. Cerebral Palsy (CP) is the MC childhood movement disorder, 1-2 cases per 1000. b. Approximately 33% have seizures, ~60% are mentally retarded. c. EEG and cognitive testing can be done to evaluate each of these respectively.

SUMMARY OF RECOMMENDATIONS FOR CHILDREN WITH CONFIRMED (VENOUS) ELEVATED BLOOD LEAD LEVELS:

a. Chelation in an asymptomatic child = (CaEDTA) IM or (DMSA, succimer) Oral. hospitalized = both b. Fluid balance is tricky; urine output is maintained because CaEDTA is renally excreted, but encephalopathy may be exacerbated with overhydration

A 15-year-old boy presents to your clinic with a 3-day history of a sore on his penis. He denies urinary frequency, change in urine appearance, or penile discharge. He reports no significant past medical history. He is sexually active and infrequently uses condoms. His examination is normal, other than a shallow, nontender ulcer approximately 1 cm in diameter on the dorsal aspect of the penile shaft. There is no lesion discharge or bleeding, but slight induration around the ulcer is noted. His urinalysis is normal. DX? Eval? tx?

a. Dx =Primary Syphilis b. Thorough H&P, focus on sexual hx, test for syphilis, and testing for concomitant STDs (gonorrhea, chlamydia, HIV) is always considered. c. Inquire about rashes: - i.e the transient, pustular rash associated with disseminated gonococcal infection or - the macular rash on the palms of patients with secondary syphilis could be identified. d. Urinary tract findings of dysuria and penile discharge are not typical for syphilis, but may be seen. e. Inguinal lymphadenopathy also is possible. f. Do VDRL or RPR, if suspicion high = MHA-TP or TP-PA testing. g. neuro symptoms = DO LP h. primary and secondary syphilis treated with one to *three weekly IM penicillin G injection(s), dependent on duration (unknown or greater than 1 year warrants three weekly injections). i. Tertiary syphilis = minimum of 10 days of IV penicillin G.

A mother brings her 2-year-old son to your clinic because she has seen over the last 2 days drops of bright red blood in his diaper with each stool and with wiping. On inspection of the rectal area you find a 7-mm linear split in the posterior midline traversing from the anocutaneous junction to the dentate line.

a. Dx: anal fissure b. Best management: Fecal Occult Blood test c. Next step: quantify blood loss, review vitals, tachy = initial sign on blood loss, hypotention = late finding. Oral PEG for consitpation, minimize dairy, increase H20 intake, avoid bulking agents (fiber/psyllium).

A 16-year old female presents with very heavy menstrual bleeding for the last 6 months. She notes her cycles are regular at 29 days, but her period lasts for 10 days ago and goes thru 10-12 pads per day. She gets dizzy when she stands up and has pallor and pale conjuctivae. A urine pregnancy test is negative and her hemoglobin is 10. Dx? How would you manage this patient?

a. Dysfunctional uterine bleeding causing anemia and orthostatic hypotension. b. Place the patient on OCPs for 3-6 months and recheck her hemoglobin in 6 weeks. c. Realize that menstrual bleeding that leads to anemia and orthostatic hypotension is not typical and warrants further investigation (pregnancy, bleeding disorders, STDs, trauma, malignancy).

which 2 usu have no fever?

a. E coli can cause bloody diarrhea but usually no fever. b. Infection with Vibrio cholera produces vomiting and profuse, watery, nonbloody diarrhea with little or no fever.

Difference b.w early and late onset sepsis?

a. Early onset: sepsis in first 6 days, 85% w.i 1 day, 5% additional day 2, remainder 4 days. b. Late onset neonatal: sepsis usually occurring after approx. 7 days but before approx. 90 days of life.

What is the step-wise approach in treating intermittent asthma? What is the step-wise approach in treating persistent asthma?

a. First step in treating intermittent asthma includes using albuterol (short acting beta-agonist) as needed, and if that doesn't work add a low dose inhaled steroid. b. First step in treating persistent asthma includes inhaled corticosteroid plus a long-acting beta agonist (plus the rescue albuterol inhaler).

An adolescent girl presents with acne on her face and shoulders. Best tx?

a. First-line therapy includes antibacterial soap, keratolytic agent (benzoyl peroxide), comedolytic agent (tretinoin), and/or topical antibiotic (erythromycin). b. Oral antibiotics (tetracycline) are a secondary option. Isotretinoin (oral tretinoin) is reserved for severe, resistant nodulocystic acne.

A 17-year-old girl presents with severe pain in the right upper quadrant and has some pain in her right shoulder. She has nausea, fever, and chills. The abdominal pain increases with movement or Valsalva activities. On physical examination, you confirm pain over the gallbladder, but also notice that she has right lower quadrant abdominal pain. Her pelvic examination is significant for discharge from the cervical os and pain upon cervical motion. Which of the following is consistent with the most likely diagnosis?

a. Fitz-Hugh-Curtis syndrome. b. This disease can be seen in both genders, MC in girls and is usually (but not always) associated with evidence of acute PID. c. The RUQ pain results from ascending pelvic infection and inflammation of the liver capsule and diaphragm. It can mimic other abdominal emergencies and must be considered in sexually active adolescents as a diagnosis of exclusion. d. caused by -Chlamydia trachomatis infection probably is more common than GC.

PLEURAL EFFUSION:

a. Fluid accumulation in the pleural space; may be a/w chest pain or dyspnea b. can be transudate or exudate depending on results of fluid analysis for protein and lactate dehydrogenase c. origins include cardiovascular (congestive heart failure), infectious (mycobacterial pneumonia), and malignant (lymphoma).

Delivery for an AGA 36-week old premie occurred 19 hours after membrane rupture--mom received no prenatal care. Baby has sudden onset of "grunting" with a RR of 60 breaths/min. His temperature is 96.5 and is BP is low. CBC shows 2500 whites and 80% bands (low white count with left shift). Dx?

a. GBS pneumonia. b. This infant has several risk factors for GBS infection: prematurity (<37wk), membrane rupture over 18hr before delivery, chorioamnionitis and no prenatal care--other factors include intrapartum fever and previous infant with GBS infection. c. EXTRA--What antibiotic would mom have received during labor to prevent GBS newborn infection? IV Ampicillin or penicillin

What is Guttate psoriasis? Nummular dematitis? PITYRIASIS LICHENOIDES CHRONICA?

a. GUTTATE PSORIASIS: A variant of psoriasis, often following a Strep or perianal infection , whereby a sudden eruption of small round or oval psoriatic lesions on the trunk, face, and proximal limbs occurs. b. NUMMULAR DERMATITIS: - Pruritic boggy or vesicular round lesions that erupt on the extremities, buttocks, and shoulders. - The lesions can be boggy, vesicular, and weepy or dry and scaly. - When chronic, lichenification can occur. c. PITYRIASIS LICHENOIDES CHRONICA: - Multiple, small (3-to 5-mm), reddish-brown papules covered with grayish scale develop on the trunk + extremities. - benign - The lesions can become vesicular, hemorrhagic, crusted, or superinfected. - By 2 to 6 wks these lesions become flat and hyper-or hypopigmented. - lubricants, topical steroids, or erythromycin

MUSCARINIC SYMPTOMS:

a. Gastrointestinal (emesis, urinary and fecal incontinence) b. respiratory (bronchorrhea, bronchospasm) c. cardiac (hypotension, bradycardia) d. tearing and drooling e. miosis.

Organisms that commonly cause early-onset sepsis colonize in the mothers UG tract and are acquired transplacentally from an ascending infection or as the infant passes thru the birth canal. What are the 4 most common organisms? NOTE: Early-onset sepsis is 0-7 days: treat with PCN + aminoglycoside. Late-onset is 7-90 days: treat with vanc + 3rd g ceph.

a. Group B strep, E.coli, Listeria, and H. flu. b. Remember that Group B strep is the most common cause of neonatal sepsis from birth to 3 months, and mortality as a result of GBS disease is 10%. c. Finally, approximately 80% of early-onset disease (septicemia, pneumonia, and meningitis) result from vertical transmission during labor. d. late onset = when infant becomes infected in postnatal period, organisms = coagulase-negative staphylococci, Staphylococcus aureus, E coli, Klebsiella sp, Pseudomonas sp, Enterobacter sp, Candida, GBS, Serratia sp, Acinetobacter sp, and anaerobes.

If PNA patient has been... a. spelunking? b. exposed to cattle? c.traveled to Southwest USA? d. exposed to stagnant water + is hyponatremic? e. has cystic fibrosis? f. has bloody sputum/fever/cough/weight loss/night sweats?

a. Histoplasma b. Coxiella c. Coccidioides d. Legionella e. Staph initially, then Pseudomonas; f. M. tuberculosis

37 week- gestation male..At birth he was lethargic and had an HR of 40. Oxygen was administered via bag and mask, and he was intubated; his HR remained at 40 bpm.

a. If the HR < 60 bpm despite PPV with 100% oxygen, then chest compressions are given for 30 seconds. b. If the HR is still < 60 bpm, then drug therapy (usually epinephrine) is indicated.

histology of HSP, IgA, HUS?

a. Immune complex deposition with immunoglobulin (Ig) A in the mesangium is seen in HSP and IgA nephropathy. b. Endothelial cell swelling with fibrin deposition is seen in hemolytic-uremic syndrome. c. crescentic glomeruli are seen in rapidly progressive glomerulonephritis.

In utero, what does the PDA do?

a. In utero, the ductus arteriosus shunts blood from the quiescent lungs---> pulmonary artery -->descending aorta. b. Failure of DA to close =allows shunting of blood from the systemic to the pulmonary circulation, causing myocardial stress, pulmonary vascular congestion, and respiratory difficulty. if not closed = risk of Infection Endarteritis or paradoxical emboli.

Symptoms of Pneumonia

a. Increased work of breathing - nasal flaring, acessory muscle use or tachypnea. b. malaise, HA, abdominal pain, nausea, emesis. c. rales/wheezing

Meperidine given to mom during labor can cause what problems? Tx

a. Infant becomes apneac and has poor respiratory effort. b. IV, IM, SQ or ET administration of naloxone.

A term female infant is born vaginally after an uncomplicated pregnancy. She appears normal but has respiratory distress when she stops crying. When crying she is pink; when not she makes vigorous respiratory efforts but becomes dusky. Which of the following is the likely explanation for her symptoms?

a. Infants = obligate nose breathers until about 4 mos b. When crying they can breathe through their mouth, but they must have a patent nose when quiet. c. Choanal atresia is Id'd by passing a feeding tube through each nostril or by identification of clouding on cold metal held under the infant's nose. d. tx: endotracheal intubation to bypasses the airway obstruction until surgical repair can be completed.

A 5-year-old girl is referred to a pediatric rheumatologist with a 4-week history of mild swelling and decreased range of motion in the left knee and right elbow. She is afebrile and appears otherwise well. Positive findings on which of the following evaluations will be most helpful in establishing her diagnosis?

a. JIA is the most common cause of uveitis in children. b. Uveitis onset may be insidious, and may be the only initial manifestation of JIA. c. The disease is more common in young girls. Slit-lamp findings include band keratopathy, posterior synechiae, and cataracts.

A 3-year old, previously healthy child now living in a home under renovation has developed seizures, abdominal pain + vomiting, constipation, "achy bones," and emotional lability. Dx? Best test to diagnose? Tx--and what BLL (blood lead level) should it be used at?

a. Lead poisoning from the lead-based paints in the old home is the cause--sometimes these kids have pica as a symptom. b. Blood lead level (BLL) should be obtained. c. Best treatment is to remove child from the source and initiate chelation therapy with Succimer/DMSA at levels above 45ug/dL (do not treat if less than 45). - screen other kids in room.

A 7-year-old has right-sided abdominal pain and fever to 102°F (38.9°C). His mother says that he has had 2 days of poor appetite and cough; he had two loose stools earlier in the day. On examination, his temperature is 101.7°F (38.7°C), his heart rate is 120 bpm, and his respiratory rate is 50 breaths/min. Breath sounds are diminished, and the abdomen is diffusely tense with hypoactive bowel sounds. Which of the following would likely lead to the diagnosis?

a. Lower lobe pneumonias can cause abdominal pain, which may be the most distressing symptom in a young patient. b. Inflammation of the diaphragm can result in an abnormal abdominal examination, which may be mistaken for the source of the child's illness. c. This child has cough, fever, tachypnea, and diminished breath sounds, which together make pneumonia the most likely diagnosis.

A mother brings in her 1-week-old son who has vomited four times over the last 24 hours. He has no fever or diarrhea. The infant is breast-feeding poorly and is "floppy" per the mother. He has had only one wet diaper in the last 12 hours. Physical examination reveals a lethargic infant who has lost 250 g since birth, with pulse of 110 bpm, dry oral mucosa, and no skin turgor. Which of the following levels should be checked after stabilization and electrolyte measurement?

a. Male infants with salt-losing CAH develop clinical symptoms similar to pyloric stenosis, intestinal obstruction, heart disease, cow's milk intolerance, and other causes of failure to thrive. b. Their genitalia appear normal. c. A serum 17αhydroxyprogesterone level typically is elevated. d. Without appropriate treatment (hydrocortisone, mineralocorticoid, and sodium supplementation), CV collapse and death may occur w/in a few weeks.

Imaging techniques

a. Meckel Scan - meckels diverticulum b. U/S or air contrast enema for intussusception c. Angiography d. tagged red blood scan for low flow bleeds.

What is routine care for child with SCD?

a. initiation of daily penicillin therapy by 2 months of age + folate by 6 months of age. b. Special vaccinations at 2 years of age—*meningococcal and the 23-valent polysaccharide pneumococcal vaccines*; additional doses of these vaccines may be required. c. Additional laboratory, radiologic, and other testing also may be indicated.

A 2-year-old girl presents with her second episode of bloody stool. Mom brings a diaper that is filled with brick-colored stool. The child has had less appetite than usual for the day but no fever, vomiting, or complaints of pain.

a. Meckel diverticulum = pouch off the ileum due to a remnant of the omphalomesenteric duct. b. 3 to 6 cm in size and located 50 to 75 cm from the ileocecal valve. c. It is often lined with endothelium that has undergone meta-plastic change that simulates gastric mucosa-->the acid that is secreted causes ulceration of the adjacent ileal mucosa. d. Symptoms of intermittent painless rectal bleeding usually appear at the age of 2 years. e. It produces 50% of all lower GI bleeds in children under the age of 2 years. f. A Meckel radionuclide scan needed for dx, but --> high false-negative rate, so laparoscopy may be needed. g. Even if the bleeding stops, surgical excision of the mucosa is often done to prevent re-bleeding,obstruction, or diverticulitis.

What kind of deficiency (micro/macro/norm) results if you have low iron? if you have acute blood loss? if you are a newborn being breast-fed by a vegan mom? if you are a newborn being fed pasteurized goat's milk?

a. Microcytic anemias include IRATS (iron deficiency, anemia of chronic disease, thalassemia, sideroblastic anemia). b. Normocytic anemia results from many things, 1 being acute blood loss. c.Macrocytic (megaloblastic) anemia results from: - Vitamin B12 and folate deficiencies - D.latum infection - breast-feeding from vegan mom - brucellosis - goat's milk (infant needs vit + mineral supplementation) - hypothyroidism, - trisomy 21. - (Note: B12 def shows neuro signs, folate def does not)

A 6-year old boy has a 1-week history of joint pain and limping but no recent trauma. He has a low-grade fever, Hepatosplenomegaly and petechiae on his face and chest. Dx? Next step?

a. Most likely diagnosis is ALL. b. Next step in evaluation is obtain a CBC with platelets and differential; bone marrow biopsy is required to confirm ALL. c. Leukemia is the most common childhood cancer (about 40% of all childhood cancers). d. Worse prognostic factors include age <1 or >10, mature T/B-cell (versus pre-B/T), WBC count >50,000, male gender, and Afr-Amer/Hispanic descents, T precursor immunophenotype.

A full-term newborn is born with ambiguous genitalia. This is a psychosocial emergency and gender assignment should aim to achieve unambiguous and sexually useful genitalia. - Most likely diagnosis? - Next step in evaluation? - What's the most important factor in determining an infant's sex assignment? - If the diagnosis is CAH, what's the treatment?

a. Most likely diagnosis is Congenital Adrenal Hyperplasia (most common = 21-beta-hydroxylase deficiency): CAH decreases cortisol production, increases 17-alpha-hp levels, and causes virilization. b. Next step in evaluation includes getting a karyotype, serum electrolyte levels and a serum 17-alpha-hydroxyprogesterone level, c. but phallic size is the most important factor in determining an infant's sex (microphallus is < 2cm stretched)! After assigning gender, intervene with psychological, surgical and hormonal treatments ASAP. d. If it is CAH, administration of hydrocortisone will inhibit both excess androgen production and further virilization.

A 3-year old boy has 20 days of high-spiking fevers associated with a faint rash on his trunk and extremities. He has a 1-day history of refusal to bear weight. His exam is significant for lymphadenopathy, organomegaly, and joint swelling. His CXR is negative, but the CBC reveals leukocytosis, thrombocytosis, and anemia. Dx? Best test? Tx?

a. Most likely diagnosis is Systemic-onset juvenile idiopathic arthritis, -The daily high-spiking fevers a/w a characteristic rash are suggestive of systemic JIA. - Arthritis may develop after other symptoms begin, even months or even years into the disease course. -For cases where arthritis first appears in the disease course, leukemia is a consideration. - There is no lab studies that are diagnostic of JRA but ordering a CBC, ESR, RF, ANA, and synovial fluid analysis are good to get. -Treatment includes NSAIDs, MTX, and corticosteroids.

Acute arsenic ingestions can cause?

a. N/V/D abdominal pain. b. The third spacing and hemorrhage in the gut can lead to hypovolemic shock. c. Cardiac symptoms include ventricular tach (QT prolongation) CHF. d. seizures, cerebral edema, encephalopathy, and coma. e. Early on, patients develop loss of DTR, paralysis, painful dysesthesias, and respiratory failure similar to Guillain-Barré syndrome. f. Fever, anemia, alopecia, hepatitis, and renal failure also can be seen.

A 3-day old Asian boy is jaundiced, breast-fed, and has a cephalohematoma. Dx? Tx? If this patient's jaundice worsened and showed signs of poor feeding, lethargy, and a poor Moro reflex--what are you considering?

a. Neonatal hyperbilirubinemia; b. Phototherapy if bilirubin is 16-18 on day two; c. Kernicterus. - This infant has several risk factors for neonatal jaundice: Asian, male gender, cephalohematoma, and breast-feeding. - You should always think PATHOLOGY if the jaundice appears < 24 hrs after birth, if bilirubin rises greater than 5mg/dL per 24 hours, and if jaundice persists >10 days.

A 13 y.o. boy has a 1-day history of fever and lethargy and was unable to be awoken this morning. In the ER, his RR is 7, HR 55, BP 60/40, and temperature 106. He has altered mental status and a purpuric rash over his trunk. What's the next step? Diagnosis?

a. Next step is to utilize the ABCs of airway management and intubate the child to secure his airway. b. The following step would be to start him on antibiotics (PCN or cephalosporin) because he likely has meningitis/sepsis caused by Neisseria meningitis, as evidenced by his purpura fulminans.

Acne lesions are categorized as inflammatory or noninflammatory: differences?

a. Noninflammatory lesions consist of open and closed comedones. b. Inflammatory lesions are characterized by the presence of papules, pustules, nodules, or cysts.

Temperature instability, tachypnea, hypotension, and bradycardia are common findings in sepsis AND in meningitis. However, there are two other signs (i.e. vascular signs) that are manifesting in shock patients, what are they? Always order what in suspected cases of shock?

a. Pallor and poor capillary refill. Always order a blood culture in suspected shock patients. b. Further differentiating PNA and shock--pneumonia is associated with more respiratory distress signs like grunting, nasal flaring, retractions, decreased breath sounds, and cyanosis.

GI bleeding:

a. Patients are admitted to the hospital for monitoring, and intravascular volume is initially restored with *isotonic saline, then packed red blood cells may be needed*. b. Frequent measurement of *Hg or Hematocrit is indicated* and bleeding can be monitored in each stool via guaiac testing if blood is not grossly visible. c. Lab eval for: PT, APTT, liver enzymes, plateletes, Cr, BUN. d. X rays in neonates to look for NEC signs (intramural air in portal venous sys)

A 10-year-old with a diffuse, slightly pruritic annular rash that developed 10 days after a single large lesion was noted on her lower back. It started with one small, scaly red area on her chest and then spread. PE reveals salmon-colored, flat, finely scaly, oval eruptions on the aforementioned areas. Diagnosis? Tx?

a. Pityriasis rosea is preceded by a "herald patch," an annular scaly, red lesion. (if child was sexually active + rash on palms and soles-> think secondary syphilis as part of differential). b. The lesions are salmon-colored and in a Christmas-tree formation following the lines of the skin. c. The cause is unknown. The rash usually lasts 6 weeks and then resolves. d. Best management: Supportive care with emollients and occasionally antihistamines if the pruritus is significant.

What age do you diagnose precocious puberty in girls and boys? Further, what do the bone age radiographs look like in precocious puberty? Treatment and goal of treatment? What age do you diagnose delayed puberty in girls and boys?

a. Precocious puberty = secondary sexual characteristics onset before age 8 in girls and 9 in boys; bone age radiographs are advanced beyond chronological age; goal of treatment is to prevent premature closure of the epiphyses, allowing the child to reach full adult growth potential-- b. GnRH agonists are used in Tx because they'll desensitize the gonadotropic releasing cells of the pituitary. Delayed puberty = no signs of puberty in girls by age 13 or in boys by age 14.

Path of syphilis: Primary and Secondary

a. Primary syphilis = painless ulceration that usually erupts on the genitalia or perianal region (oropharyngeal ulcer also possible) within 2 to 3 weeks of transmission, and spontaneously resolves over 4 to 6 week b.If untreated, secondary syphilis may develop w/i 2 to 3 months, with malaise, fever, lymphadenopathy, and a stereotypical rash (macular to papular lesions often found on the palms and soles), or nondescript rash that may mimic an allergic dermatitis or a viral exanthem. c. Tertiary has problems with = the skin (gummas), cardiovascular system (aortic aneurysm), or central nervous system (neurosyphilis with possible meningitis, seizures, or musculosensory deficits).

How do you use pulse oximetry to dx R--> L shunt?

a. Pulse ox can be used to measure the oxygen saturation of the tissues that are perfused by the portion of the aorta that is PROXIMAL to the ductus (the right handor an ear lobe) and then is compared to tissues perfused by the portion of the aorta that is distal to the ductus (the lower extremity). b. If there is a difference of more than 3% to 5%, then there may be a *right-to-left* shunt across the ductus.

HEMOLYSIS:

a. Rapid breakdown of RBCs. b. Clinical and laboratory findings might include a rapid rise of serum bilirubin level (>0.5 mg/dL/h), anemia, pallor, reticulocytosis, and hepatosplenomegaly.

An 18-year old mom with no prenatal care delivers a baby via emergency C-section. The baby is 4500g, dusky gray, not breathing, poor tone and has a HR of 100. Dx? Next step?

a. Respiratory distress in an infant of a diabetic mother (IDM). b. Next step is ABCs and to check for hypoglycemia. c. Diabetes (>100mg/dL glucose in a fasting state) occurs in 3% of pregnancies. d. Women are screened for gestational diabetes between 24-28 weeks.

What are some sequelae of gestational diabetes? part 2

a. Respiratory distress syndrome b. polycythemia with hyperviscosity syndrome c. hypocalcemia d. hypomagnesemia e. hyperbilirubinemia f. Infants can be small for gestational age if placental insufficiency is present.

GBS signs?

a. Respiratory signs (apnea, grunting respirations, tachypnea, or cyanosis) are the initial clinical findings

If you are suspecting a shigella or salmonella gastroenteritis and you order a CBC, how can you differentiate the two? How do you treat gastroenteritis?

a. Salmonella infection usually results in a mild leukocytosis. b. Shigella will have a normal WBC count but a remarkable left shift, i.e. more bands than PMNs. c. Treatment focuses on fluid and electrolyte balance correction. d. Antibiotics are not necessary for Salmonella as the don't shorten the disease course, and they even increase the risk for HUS. e. Shigella is also self-limited but Bactrim does shorten the disease course.

Complications of SCD?

a. Sepsis b. Acute Chest c. Splenic enlargement --> can cause CV collapse. d. Splenic auto-infarction --> increase odds of encapsulated organisms. e. 10% children have acute stroke (paresis, aphasia, seizure, CN palsy, HA, coma) --> Transcranial Doppler U/S is often recommended to ID those with increased flow velocity in large cerebral vessels (high risk for stroke). f. Aplastic crisis (pallor, fatigue, lethargy, low Hg, low Retic) g. Priapism ( > 3 to 4 hrs --> urologist needed)

CLINICAL PEARLS:

a. Subdural hemorrhage: - MC in children younger than 1 year and in the supratentorial space; - seizures and retinal hemorrhages and increased ICP is typical. b. Epidural hemorrhages: -MC in older children and adults and in the infratentorial space. - Fewer than 25% of patients have seizures; retinal hemorrhages are uncommon. c. Mortality with subdural hemorrhage is generally less than that seen with epidural hemorrhage, but long-term morbidity is more significant with subdural injury because the brain parenchyma is more often involved.

A mother brings in her 15-year-old daughter because she has never started her periods. She otherwise is healthy and takes no medications. Her past medical history is unremarkable except for inguinal hernia repair as an infant. Family history is unremarkable. She is at the 75th percentile for height and weight, has Tanner stage IV breast development, and no pubic or axillary hair development. Her anogenital examination reveals a short, pocketlike vaginal opening. Which of the following is the most likely explanation for her amenorrhea?

a. Testicular feminization results from decreased androgen binding to target tissues or androgen insensitivity. b. Patients have 46,XY karyotypes, yet appear as phenotypically normal females with a short or atretic vagina.

Vaccine ages for SCD:

a. The 23 valent polysaccharide pneumococcal vaccine is initiated at 2 years of age b. Conjugate pneumococcal vaccine is given at 2,4,5 mos of age.

A term male is delivered vaginally, he is noted to have a scaphoid abdomen, cyanosis, and respiratory distress. Heart sounds are heard on the right side of the chest, and the breath sounds seem to be diminished on the left side. tx?

a. The case describes diaphragmatic hernia. b. As a result of herniated bowel in the chest, often have pulmonary hypoplasia. c. Bag-and-mask ventilation will cause accumulation of bowel gas (which is located in the chest) and further respiratory compromise. so, endotracheal intubation is the best course of action.

X ray and ECG findings of ASD?

a. The chest radiograph reveals an enlarged right atrium, right ventricle, and pulmonary artery and increased pulmonary vascularity; b. ECG shows right ventricular hypertrophy and sometimes right-axis deviation.

What is the mechanism behind macrosomia in infants of diabetic mothers?

a. The fetal pancreas becomes functionally significant after week 26 in gestation. b. Hyperglycemia in the mom causes the baby to produce a lot of insulin as a response. c. Insulin stimulates growth hormone, increased adipose tissue deposition, and increased glycogen deposition in many organs (except the brain--thus head circumference isn't affected).

What is the most common cause of death in kids age 1 week to 1 year? What season of the year is it most common? In what ethnicities? In what sleep position? What gender? Term or premie?

a. The most common cause of death in kids age 1 week to 1 year is SIDS. peak incidence b/w 2 and 4 months. b. Risk factors include: winter months, African-Americans and Native Americans, prone sleep positions, males, and premies, bed sharing, soft surface, postnatal exposure to smoke, over-heating, LBW. Apnea is no longer considered a risk factor, however being a sibling of a SIDS victim is.

A 3-year old previously healthy child was helping her father spray for bugs when she suddenly developed salivation, tearing, respiratory distress, and urine incontinence. Dx? Tx?

a. The muscarinic symptoms seen suggest organophosphate poisoning (MOA: irreversibly binds/inhibits cholinesterase). b. Poisoning occurs across skin or mucous membranes, inhalation or ingestion. c. The child should be decontaminated (take poisoned clothes off and wash skin) and should receive atropine + pralidoxime. Atropine is an anticholinergic and pralidoxime reactivates cholinesterase.

Clinical pearls of meningitis:

a. The typical meningitis presentation in older children consists of fever, headache, and nuchal rigidity. b. Nuchal rigidity is not a reliable finding of meningitis until 12 to 18 months of age. Pneumococcal disease (including meningitis) is more common in patients with functional or anatomic asplenia. c. Approximately 1/3 of meningitis patients have a seizure at some point in the disease. d. Typical CSF findings of bacterial meningitis include elevated protein level, reduced glucose concentration, and several hundred to thousands of white blood cells per cubic millimeter.

A term, 3700-g infant is born vaginally without complications and has uneventful immediate neonatal course. - At 2 weeks of age, a *II/VI systolic murmur is noted in the mitral area that radiates to the back*. A similar murmur is noted in the *right axilla*. - The infant is pink and breathing easily, and the nurses notes show that he has been taking 30 cc of formula approximately every 2 hours. Initial management should include which of the following?

a. This infant has *peripheral pulmonic stenosis*, a benign childhood murmur. b. Other benign childhood murmurs are - venous hum (a low-pitched murmur heard at the sternal notch only when the child is upright) and c, Still vibratory murmur (a high-pitched "musical" systolic murmur heard best at the left sternal border in the supine position).

A previously healthy term infant suddenly develops respiratory distress on day 3 of life. An echocardiogram reveals coarctation of the aorta. Which of the following is the most appropriate treatment for immediate stabilization of this infant?

a. This infant's symptoms started when his ductus arteriosus began to close. b.Prostaglandin therapy can reverse this process in the short-term. c. Surgery or catheterization techniques provide definitive repair.

TX of acute post strep glomerulonephritis is?

a. Treatment is generally supportive. b. Fluid balance is crucial; c. diuretics, fluid restriction, or both may be necessary. d. Sodium and potassium intake may require restriction. e. HTN - use calcium-channel blockers. f. Strict bed rest and corticosteroid medications are not helpful. Dialysis is rarely required

Physical findings of ASD?

a. a widely split second heart sound that does not vary with respiration ("fixed splitting") b. and a systolic murmur at the left upper and midsternal borders caused by high-volume blood flow from the right ventricle into the normal pulmonary artery; the murmur is not blood flowing across the ASD itself. c. A lower left sternal border diastolic murmur produced by increased flow across the tricuspid valve may be present.

Beyond the newborn period and through approximately 5 years of age, viral pneumonia is common: such as?

a. adenovirus b. rhinovirus c.RSV d. influenza, and parainfluenza are possibilities. e. Bacterial etiologies include pneumococcus and nontypeable Haemophilus influenzae.

Pathophy of asthma?

a. airway inflammation in asthma = result of mast cell activation b. IgE response to environmental triggers = vasodilation, increased vascular permeability, smooth-muscle constriction, and mucus secretion.

Approximately 60% of pediatric pneumonias are ____ in origin

a. bacterial: pneumococcus Mc b. Viruses (respiratory syncytial virus [RSV], adenovirus, influenza, parainfluenza, enteric cytopathic human orphan [ECHO] virus, and coxsackie virus) run a close second.

HYPOGLYCEMIA:

a. blood glucose level less than 40 mg/dL symptoms: lethargy, listlessness, poor feeding, temperature instability, apnea, cyanosis, jitteriness, tremors, seizure activity, and respiratory distress

Prior to emergent surgical intervention, the initial management of patients with malrotation and volvulus includes?

appropriate evaluation of fluid status as patients may have significant fluid loss with electrolyte abnormalities. Ill appearing infant: placement of a NG tube to aid GI decompression, and initiation of parenteral antibiotic, in order to address potential sepsis are indicated.

As adults, males with Klinefelter syndrome develop:

gynecomastia, sparse facial hair, and azoospermia, increased breast CA and heme malignancies.

Patients with malrotation and either partial or intermittent volvulus may present with?

may present with recurrent abdominal pain or lymphatic congestion leading to failure to thrive because of malabsorption or chylous ascites.

Fragile X syndrome:

mc form of inherited mental retardation - mc in boys -diagnosed in patients with mental retardation (particularly boys) who have macrocephaly, long face, high arched palate, large ears, and macroorchidism after puberty.

Intubation and suctioning of a neonate below the vocal folds hints at?

meconium aspiration- would be done in delivery room.

A 13-year-old adolescent female complains of dry cough, slight fever, and fatigue over the past 2 weeks. She noted increased chest congestion and coughing yesterday when walking outside in the cold air. She denies nasal congestion, rhinorrhea, emesis, or diarrhea. What is walking pneumonia?

mycoplasmal infection: Hemolysis occurs as antibodies attach to red blood cells, prompting reticulocyte production. If necessary, nasopharyngeal aspirate for PCR or measurement of cold agglutinins may help aid in the diagnosis.

who gets prophylactic abs?

previous history of: - endocarditis - prosthetic valve or material for repair - heart transplant patients - severe or partially repaired cyanotic congenital heart defects.

Administration of phenobarbital induces glucuronyl transferase, thus _______ neonatal jaundice.

reducing

CF: Recurrent sinusitis is uncommon in young children because...

their nasal passages are not fully pneumatized; this girl likely was incorrectly diagnosed or has an underlying, predisposing condition for this problem.

What should be avoided in DS?

until unilateral lateral cervical flexion-extension films confirm normal anatomy - contact sports that may result in forceful flexion of the neck should be avoided.

A 2-year old former premie had post-delivery complications of necrotizing enterocolitis, where a small portion of her intestine including her ileocecal valve was removed. Mom said she is growing fine, but notes her daughter's pallor. CBC shows a Hb of 7 and a MCV of 110. What's the mechanism of this child's anemia? dx?

• Most likely cause: Vitamin B12 deficiency secondary to terminal ileal resection and compromised intestinal absorption. • Treatment: Monthly intramuscular vitamin B12 supplementation. Remember "FeFoCo" for small intestine reabsorption --> Co is cobalamin = B12, thus also ADEK absorbed here. Iron mostly absorbed in the duodenum, and folate in the jejunum.

A 2-year-old healthy girl with several weeks of nocturnal perianal and perineal pruritus. On examination, the perianal area is red and irritated; the anal sphincter tone is normal, and you find no evidence of penetrating trauma. Dx? Management?

• Most likely diagnosis: Infection with Enterobius vermicularis (pinworms). • Confirm the diagnosis: Cellophane tape test with microscopy to identify pinworm eggs Best management: Mebendazole, pyrantel pamoate, or albendazole in a single dose, treating the entire family.


Conjuntos de estudio relacionados

Fluid, Electrolyte, and Acid Base Regulation Assessment

View Set

Pharm Pain - Opioid Analgesics (ch 10)

View Set

CH 1: Completing the Application, Underwriting, and Delivering the Policy

View Set

Chapter 10, 11, 12, 13, 14, 17 canvas and chapter quizzes

View Set

US History - Chapter 10 Study Guide

View Set